You are on page 1of 31

Practice Questions 1

A1 C1 3 D. take time now to allow the patient to express her


1. A forty-eight year-old female sustained a fracture to her left fears and frustrations.
shoulder. Treatment is proceeding well except that with left
shoulder flexion you notice the scapula protract and A6 C5 3
elevate early and it continues to move excessively. 6. Your examination reveals muscle spasms of the deep hip
Physical therapy intervention should emphasize: rotators, which are compressing the sciatic nerve and
A. glenohumeral mobilization and strengthening of producing pain in the posterior hip region. The MOST
scapular stabilizers to regain normal scapulohumeral effective setting of ultrasound in this case is:
movement. A. 1 MHz continuous at 1.0 W/cm2.
B. glenohumeral mobilization, and strengthening of the B. 1 MHz pulsed at 1.0 W/cm2.
rotator cuff muscles to regain muscle balance. C. 3 MHz continuous at 1.0 W/cm2.
C. scapulothoracic mobilization and strengthening of the D. 3 MHz pulsed at 1.0 W/cm2.
pectoralis major and minor muscles to regain normal A7 C2 2
scapulohumeral rhythm. 7. A 17 year-old patient is recovering from a complete spinal
D. stretching of scapular stabilizers and strengthening of cord injury, at the level of L2. The expected outcome in this
the pectoralis major and minor muscles to regain case would MOST likely include:
muscle balance. A. a spastic or reflex bladder.
B. greater loss of arm function than leg function with
A2 C8 1 early loss of pain & temperature sensation.
2. A 59 year-old ex-machinist demonstrates significant age- C. loss of motor function, pain & temperature sensation
related hearing loss, presbycusis. When trying to below the level of the lesion with light touch,
communicate with this patient you would NOT suspect: proprioception and position sense preserved.
A. bilateral hearing loss, at all frequencies since he has D. some recovery of function since damage is to
had this problem for a number of years. peripheral nerve roots.
B. decreased language comprehension.
C. poor auditory discrimination. A8 C7 3
D. unilateral hearing loss. 8. A physical therapist is gait training a patient with left
hemiplegia. The patient’s new AFO arrives, but the
A3 C3 3 therapist is overwhelmed with too many patients and asks
3. You are supervising the exercise of cardiac rehabilitation the physical therapy student to take over. This is the
outpatient class on a very hot day, with temperatures student’s first affiliation (second day) and she has never
expected to be above 90 degrees F. Your class is performed an orthotic checkout for a patient with an AFO.
scheduled for 2 p.m. and the facility is not air conditioned. The supervising therapist will be in the same vicinity
The strategy that is unacceptable is to: treating other patients. This task should:
A. change the time of the exercise class to early A. be considered an advanced task, but allowable for
morning or evening. the student to perform as a good learning experience.
B. decrease the exercise intensity by slowing the pace B. be designated as a more advanced task and more
of exercise. appropriately delegated to another physical therapist.
C. increase the warm-up and cool-down periods to equal C. be designated as a routine task and appropriate for
the total aerobic interval in time. the student who could call out to the supervisor if
D. make the exercise intermittent by adding rest cycles. problems arose.
D. not be completed now and the patient sent back to
A4 C6 1 his room.
4. A patient has a transtibial amputation and has recently
been fitted with a PTB socket. During your initial prosthetic A9 C1 2
checkout, you instruct the patient to walk several times in 9. A patient has been diagnosed with acute synovitis of the
the parallel bars. You then have him sit down and take the temporomandibular joint. Early intervention should focus
prosthesis off. You inspect the skin. You would expect no on:
redness in the area of the: A. application of an intraoral appliance and
A. anterior tibia, tibial crest, and fibular head. phonophoresis.
B. patellar tendon and tibial tuberosity. B. instruction to eat a soft food diet and phonophoresis.
C. medial tibial and fibular plateaus. C. joint mobilization and postural awareness.
D. distal end of the residual limb. D. temporalis stretching and joint mobilization.

A5 C4 2 A10 C1 3
5. A patient who is terminally ill with cancer is in tears, unable 10. During a cervical spine examination you observe restricted
to cope with the changes in her life and with her current left rotation of the C7-T1 spinal level. After stabilizing the
hospitalization. You have a referral for gait training so she thoracic spine, your hand placement for mobilization to
can be discharged to home under hospice care. The BEST improve left rotation should be at the:
approach for you to take is: A. posterior left C6 articular pillar.
A. ask the patient questions so you can gain a detailed B. posterior left C7 articular pillar.
history. C. posterior right C7 articular pillar.
B. encourage denial so she can cope better with her D. T1 spinous process.
life’s challenges.
C. ignore her tears and focus on her therapy but in a A11 C8 1
compassionate manner.

’05 Sullivan A
2 Practice Questions

11. An appropriate fine motor behavior that should be C. tell her she’s over-reacting, she has to get on with her
established by 9 months of age would be the ability to: therapy.
A. build a tower of 4 blocks. D. tell the nurse case manager to monitor her behavior.
B. hold a cup by the handle while drinking.
C. pick up a raisin with a fine pincer grasp. A17 C5 3
D. transfer objects from one hand to another. 17. A 23 year-old college volleyball player complains of
moderate pain resulting from a left hamstring strain four
A12 C2 1 weeks ago. The focal point of pain and tightness is noted
12. A patient with a 10 year history of Parkinson’s disease has where a hematoma developed initially. The specific
been taking L-dopa for the last 5 years. He presents in your massage technique that would be MOST beneficial in this
clinic with deteriorating function. He is not longer able to case is:
transfer or walk independently. During the oral interview, A. friction.
you observe facial grimacing with twitching of the lips and B. kneading.
tongue protrusion. He appears restless, with constant C. stroking.
dancing, athetoid-like movements of his legs. Your BEST D. tapotement.
course of action is to:
A. complete your evaluation focusing specifically on his A18 C8 2
main problems of rigidity and bradykinesia. 18. A six year-old boy born with myelomeningocele at the L2
B. document your observations and refer him back to his level is referred for physical therapy treatment at home. In
physician for evaluation of possible dopamine toxicity. determining the physical therapy plan of care, it would NOT
C. talk to his wife to see if he is taking any drugs with be appropriate to emphasize:
hallucinogenic effects. A. gait training with a reciprocating gait orthosis.
D. use isokinetic dynamometry to assess his inability to B. transfer training from floor to wheelchair.
generate torque output during fast movements. C. upper extremity strengthening with weights.
A13 C7 3 D. vigorous range of motion of the lower extremities.
13. A patient who is to undergo surgery for a chronic shoulder
dislocation asks you to explain the rehabilitation following a A19 C1 1
surgical reconstructive procedure he is scheduled to 19. During your examination of a patient who complains of
undergo. Your BEST response is to: back pain you found pain with end range AROM into hip
A. explain how patients typically respond to the surgery flexion, abduction and external/lateral rotation. The
and outline the progression of exercises. structure most likely causing the patient’s pain is the:
B. explain in detail about the surgical procedure. A. hamstring muscle.
C. refer the patient to a physical therapy clinical B. hip joint.
specialist who is an expert of shoulder reconstructive C. piriformis muscle.
rehabilitation. D. SI joint.
D. tell the patient to ask his surgeon for this information.
A20 C7 2
A14 C3 2 20. A therapist wants to compare frequencies of carpal tunnel
14. Which of the following is NOT an appropriate reason to syndrome occurring in different groups of individuals:
terminate a maximum exercise tolerance test for a patient assembly line workers and computer programmers. The
with pulmonary dysfunction? MOST appropriate statistical tool to use for analysis of the
A. ECG monitoring reveals diagnostic ischemia. data is:
B. PaO2 decreases 20 mmHg. A. chi square test.
C. patient reaches age-predicted maximal heart rate. B. normal distribution curve.
D. patient states he is maximally short of breath. C. simple one-way ANOVA.
D. t test.
A15 C6 3
15. A patient with a complete T10 paraplegia is receiving his A21 C7 1
initial ambulation training. He has received bilateral Craig- 21. In a research study in which there is a skewed distribution
Scott knee-ankle-foot orthoses and is being trained with with extreme scores on a balance measure that deviate
axillary crutches. Since a reciprocal gait pattern is from the performance of the total group, the MOST
problematic for him, the BEST initial gait pattern to teach accurate representation of central tendency is:
him is: A. mean.
A. four-point. B. median.
B. swing-through. C. mode.
C. swing-to. D. standard deviation
D. two-point.
A22 C2 1
A16 C4 2 22. A 54 year-old factory worker injured his right arm in a
16. After mastectomy, a patient cannot accept the loss of her factory press with damage to the ulnar nerve at the elbow.
breast. She reports being weepy all the time with loss of A diagnostic EMG was performed with evidence of
sleep. She is constantly tired and has no energy to do spontaneous fibrillation potentials. In this case, the physical
anything. The BEST action you can take is: therapy plan of care should consider that:
A. observe her closely for possible suicide. A. axonotmesis is occurring.
B. request her primary physician to refer her for B. denervation atrophy has occurred.
psychological evaluation.

’05 Sullivan A
Practice Questions 3

C. reinnervation is complete. first floor apartment. You have determined her ambulation
D. reinnervation is in process. endurance to be only up to 15 feet, not enough to allow her
to get from her bed to the bathroom (a distance of 20 feet).
A23 C6 3 You recommend:
23. A 67 year-old patient is recovering from a left CVA resulting A. a live-in nurse (24 hour coverage) until her condition
in severe right hemiplegia. Additionally, he has a large improves.
diabetic ulcer on his left foot with pitting edema, requiring B. a skilled nursing facility placement until her
elevation of that extremity. The MOST appropriate endurance increases.
wheelchair prescription for this patient would be a: C. environmental changes, a bedside commode, and
A. hemiplegic chair. referral for home health services.
B. lightweight active duty wheelchair. D. postponing her discharge until she can complete the
C. one-arm drive chair. needed 20 feet.
D. powered wheelchair with joystick.
A29 C2 3
A24 C3 1 29. You have determined a patient with a right CVA that you
24. A 14 year-old boy with advanced Duchenne muscular are currently treating has a profound deficit of
dystrophy is administered a pulmonary function test. The homonymous hemianopsia. The BEST initial strategy to
value that is UNLIKELY to show any deviation from normal assist the patient in compensating for this deficit is to:
is: A. make the patient aware of his deficit and teach him to
A. FEV1. turn his head to the affected left side.
B. functional residual capacity. B. place items, eating utensils on his left side.
C. total lung capacity. C. provide constant reminders, printed notes on his left
D. vital capacity. side, telling him to look to the left.
D. rearrange his room so while in his bed his left side is
A25 C6 3 facing the doorway.
25. A 29 year-old woman fractured her right midtibia in a skiing
accident three months ago. After cast removal, a severe A30 C2 2
foot drop was noted. The patient desires to try electrical 30. A patient with multiple sclerosis is agitated, irritated, and
stimulation orthotic substitution. You would set up the tired during treatment. Your BEST response as the P.T. is
functional electrical stimulation to contract the appropriate to:
muscles during: A. begin pool therapy to promote relaxation.
A. foot flat. B. ignore these behaviors and continue treating.
B. push off. C. reduce the exercise intensity and provide relaxation
C. swing phase. strategies.
D. toe off. D. treat the patient in a cool environment.

A26 C4 2 A31 C7 3
26. You have a small area of dermatitis on the back of your 31. A 72 year-old medically stable individual requires custodial
hand with moderate exudate. You are scheduled to treat a care in the home. She is severely disabled with rheumatoid
patient with HIV for management of an open wound. You arthritis and is in a great deal of pain. She presents with
should: significant deformities which limit her functional abilities
A. continue with treatment as scheduled but wash your and is dependent in all basic activities of daily living. A
hands thoroughly before and after. recent exacerbation of her disease has left her bed-bound
B. double glove and treat as scheduled. for the past 2 weeks. Appropriate physical therapy services
C. refuse to treat that patient. would be covered by:
D. use sterile precautions with mask and gloves. A. Medicaid.
B. Medicare.
A27 C1 2 C. Medigap policies.
27. You are a home therapist treating a patient who underwent D. only by HMO or private insurance policies.
a total hip replacement four weeks ago. You notice that the
patient arches his lumbar spine in supine. He states that it A32 C3 2
is uncomfortable and doesn’t remember having the 32. A patient with a purulent venous insufficiency ulcer near the
problem before. The patient is unable to maintain a left medial malleolus is seen at home by a physical
comfortable supine position due to: therapist. The MOST important goal/intervention for the
A. poor abdominal strength. therapist to try to achieve with this patient is:
B. tight hamstrings muscles. A. daily changes of elastic wraps to reduce fluid buildup.
C. tight iliopsoas muscle. B. daily warm water baths to improve circulation.
D. tight piriformis muscle. C. increase ambulation endurance to hasten wound
healing.
A28 C8 3 D. instruct in proper dressing changes and wound care.
28. Your 102 year-old patient has been hospitalized for the A33 C6 1
past three days with an undisclosed ailment. After running 33. Your patient is having difficulty bearing weight on the left
numerous tests and finding nothing to explain her leg. She is unable to advance the tibia forward and
increasing weakness and fatigue, the physicians are being abbreviates the end of the stance phase on the left going
pressured to discharge her tomorrow. She lives alone in a directly into swing phase. The MOST likely cause of her
problem is:

‘05 Sullivan A
4 Practice Questions

A. hip extensor weakness. B. equipment maintenance.


B. spasticity of the anterior tibialis muscle. C. housekeeping supplies.
C. spasticity or contracture of the plantarflexors. D. long distance telephone calls.
D. weakness or contracture of the dorsiflexors. A39 C4 1
39. You are treating a patient with active hepatitis B infection.
A34 C1 1 Transmission of the disease is best minimized if you:
34. With a traction injury to the anterior division of the brachial A. avoid direct contact with any part of the patient.
plexus you would expect to see weakness of the elbow B. have the patient wear a gown and mask every time
flexors, wrist flexors and forearm pronators. You would also your are in the room.
expect to find additional weakness in: C. have the patient wear gloves to prevent him from
A. forearm supination. touching you.
B. lateral rotation of the shoulder. D. wear gloves if there is direct contact with blood or
C. thumb abduction. body fluids.
D. wrist extension.
A40 C5 3
A35 C8 3 40. A wrestler complains of pain (7/10) and limited range of
35. A 95 year-old has recently been admitted to your skilled motion of the right shoulder as a result of chronic overuse.
nursing facility following a fall-related injury (fractured hip You elect to use procaine hydrochloride iontophoresis as
with open reduction, internal fixation). Since she lived alone part of your intervention for this patient’s problems. To
on the second floor, she was unable to return home. She is administer this substance, it would be appropriate to use:
extremely agitated over her placement here and A. continuous biphasic current with the medication under
demonstrates early signs of dementia. She tells you “leave the anode.
me alone, I just want to get out of here!” An important B. continuous monophasic current with the medication
approach to take while working with this client is to: under the anode.
A. be calm and supportive, using only one or two level C. continuous monophasic current with the medication
commands. under the cathode.
B. establish the rules: tell her exactly what she is to do D. interrupted monophasic current with the medication
while you are with her. under the cathode.
C. use gesture or sign language to communicate with
her. A41 C1 2
D. promise her anything as long as she gets up and 41. A contraindication to initiating joint mobilization on a patient
walks for you. with chronic pulmonary disease may include:
A. concurrent inhalation therapy.
A36 C8 1 B. functional chest wall immobility.
36. A 72 year-old woman is being treated for depression C. long term corticosteroid therapy.
following the death of her husband. She is currently taking D. reflex muscle guarding.
antidepressant medication (tricyclics) and has a recent
history of a fall. You suspect the precipitating cause of the A42 C8 3
fall can be attributed to side effects of her medication 42. Long term care for institutionalized elderly who have
resulting in: reduced their financial resources or qualify for low-income
A. cardiac arrhythmias status is typically funded by:
B. dyspnea. A. Health Maintenance Organizations.
C. hyperalertness. B. Medicaid.
D. postural hypotension. C. Medicare.
D. Social Security Administration.
A37 C3 1
37. An 82 year-old patient and his caregivers should A43 C3 1
understand the common side effects of the medication that 43. A 45 year-old computer programmer, with no significant
he is taking. He is continually in and out of congestive heart past medical history, presents to the emergency room with
failure and has been taking digitalis (Digoxin) to improve complaints of fever, shaking chills and a worsening
his heart function. You will know he and his caregivers productive cough. He has chest pain over the posterior
understand the adverse side effects of this medication if base of his left thorax which is made worse on inspiration.
they tell you they will contact the patient’s physician if he An anterior-posterior X-ray shows an infiltrate on the lower
demonstrates: left thorax at the posterior base. This patient’s chest pain is
A. confusion and memory loss. MOST likely caused by:
B. involuntary movements and shaking. A. angina.
C. slowed heart rate. B. infected pleura.
D. weakness and palpitations. C. inflamed tracheobronchial tree.
D. trauma to the chest.
A38 C7 3
38. The Director of Physical Therapy from a large teaching A44 C7 3
hospital is asked to develop an operating budget for the 44. Equipment safety is essential in all physical therapy clinics.
upcoming fiscal year. The item that would NOT be included Regularly scheduling equipment maintenance programs to
in an operating budget is: ensure that all equipment is calibrated, lubricated, and
A. a treadmill purchase. adjusted according to manufacturer’s guidelines is an

’05 Sullivan A
Practice Questions 5

important element for patient and staff safety. All of the etiology. An initial intervention for this patient would NOT
following procedures should be followed to ensure safety include:
EXCEPT: A. limits of stability re-education including postural sway
A. conducting educational sessions for staff regarding training.
the indications and contraindications for all B. practice in maintenance of a wide base of support
equipment. during gait and turns.
B. documenting all preventive maintenance and keeping C. sit-to-stand and stand-to-sit activity training.
this information on file. D. tandem walking and single limb stance.
C. supervising new staff and students in the use of all
newly purchased equipment. A51 C6 3
D. training all staff to do simple repairs on all electrical 51. A 38 year-old patient with spinal cord injury is being
equipment if a breakdown should occur. discharged home after a 2 month course of rehabilitation.
In preparation for discharge, you and your team visit the
A45 C4 1 home and find he has 3 standard height steps going into
45. A 92 year-old woman presents with hot, red, and his home. A ramp will have to be constructed for his
edematous skin over the shins of both lower extremities. wheelchair. The recommended length of his ramp should
She also has a mild fever. The MOST likely cause of her be:
symptoms is: A. 120 inches (10 feet).
A. cellulitis. B. 192 inches (16 feet).
B. dermatitis. C. 252 inches (21 feet).
C. herpes simplex infection. D. 60 inches (5 feet).
D. scleroderma.
A52 C3 1
A46 C5 2 52. Following pneumonectomy, an expected change in a
46. A 10 year-old presents with pain and limited ROM following patient’s status would NOT include:
surgical repair of the medial collateral ligament and anterior A. asymmetrical breathing.
cruciate ligaments. The modality that would be B. decreased breath sounds.
CONTRAINDICATED in this case is: C. deviated trachea.
A. interferential current. D. increased tidal volume.
B. shortwave diathermy. A53 C3 1
C. transcutaneous electrical stimulation. 53. An older person with diagnosis of congestive heart failure
D. ultrasound. (CHF) should recognize the symptoms of exertional
intolerance. You will know this client is properly informed if
A47 C6 3 she can tell you these symptoms are:
47. Hypertrophy is the muscular response to strength training. A. dizziness, visual blurring especially with turns and
This can be expected to occur following at least: quick movements.
A. 1-2 weeks of training. B. overwhelming weakness with difficulty in standing up
B. 2-3 weeks of training. and walking.
C. 3-4 weeks of training. C. severe, uncomfortable chest pain with shortness of
D. 6-8 weeks of training. breath.
D. shortness of breath at rest and with limited activity,
A48 C1 3 and sudden weight gain
48. A diagnosis of bicipital tendinitis has been made following
an evaluation of a patient with shoulder pain. The BEST A54 C6 3
shoulder position to expose the tendon of the long head of 54. You are instructing a physical therapy student in proper
the biceps for application of phonophoresis would be: positioning to prevent the typical contractures in the patient
A. abduction. with a transtibial amputation. You stress:
B. external/lateral rotation and extension. A. maximize out-of-bed time with sitting in a chair.
C. horizontal adduction. B. position in prone lying and sitting with full knee
D. internal/medial rotation and abduction. extension.
C. position in prone-lying with slight knee flexion.
A49 C7 3 D. position in supine with a small pillow under the knee.
49. A patient is unable to bring her foot up on the next step
during a training session on stair climbing. Your BEST A55 C4 2
course of action to promote active learning is to have the 55. You receive a referral to ambulate a patient who is insulin
patient: dependent. In a review of her medical record, you notice
A. balance on the stairs while you passively bring the her blood glucose level for that day is 310 mg/dL. Your
foot up. BEST course of action is to:
B. practice marching in place in the parallel bars. A. ambulate the patient as planned but monitor closely.
C. practice standing-up from half-kneeling. B. postpone therapy and coordinate with the nurse
D. step up onto a low step while in the parallel bars. regarding insulin management and exercise.
C. refrain from ambulating the patient, reschedule for
A50 C2 3 tomorrow.
50. A 76 year-old patient exhibits impaired balance. A D. talk to the nurse about seeing the patient later on that
diagnostic work-up has failed to reveal any specific day.

‘05 Sullivan A
6 Practice Questions

A56 C8 3 A62 C3 2
56. A physical therapy plan of care for a newborn with Erb- 62. A 75 year-old patient with peripheral vascular disease has
Klumpke Palsy would NOT include: been referred for conditioning exercise. He demonstrates
A. age appropriate movements of the upper extremity. moderate claudication pain in both legs following a 12
B. gentle ROM after immobilization. minute walking test. The MOST appropriate exercise
C. partial immobilization of limb across abdomen. frequency and duration for this patient is:
D. splinting the shoulder in abduction and internal A. 2 times/week, BID 20 minutes/session.
rotation. B. 3 times/week, 30 minutes/session.
C. 3 times/week, 60 minutes/session.
A57 C1 3 D. 5 times/week, BID 10 minutes/session.
57. A six month-old child was referred to physical therapy for
right torticollis. The MOST effective method to stretch the A63 C1 1
muscle is by positioning the head and neck into: 63. The radiographic view shown in the diagram that
A. extension, left side-bending, and right rotation. demonstrates the observed spinal defect is: Twomey L,
B. extension, right side-bending, and left rotation. Taylor J (2000) Physical Therapy of the Low Back, 3rd ed.
C. flexion, left side-bending, and left rotation. Philadelphia, Churchill Livingstone, Figure 7-1, page 204,
D. flexion, right side-bending, and left rotation. with permission.
A. frontal.
A58 C7 3 B. lateral.
58. A student you are supervising is on final affiliation following C. oblique.
completion of academic training. He is overheard D. posterolateral.
discussing a patient’s history in the elevator. When you
later point this out to the student, he tells you he was A64 C4 2
unaware of any hospital policy regarding confidentiality. 64. A physical therapist is working with a patient with
Your BEST analysis of this situation is that: metastatic breast cancer who has been told that she has
A. compliance was not a realistic expectation since he only months to live. She is quite angry and disruptive
just arrived at this facility. during therapy. What is the MOST appropriate intervention
B. he should be expected to value patient confidentiality. for this patient?
C. now that he is aware of confidentiality restrictions he A. allow the patient to express her anger while
should do better the next time. refocusing her on effective coping strategies.
D. since this is not strictly part of the professional code B. forbid all expressions of anger as she is only hurting
of ethics you should not reasonably expect him to herself.
demonstrate adherence to this concept. C. provide honest, accurate information about her illness
and rehab plan of care.
A59 C7 2 D. provide opportunities for the patient to question her
59. A therapist is performing clinical research in which a impending death but limit all expressions of anger.
specific myofascial technique is applied to a patient with
chronic back pain. She is using a single case experimental A65 C1 1
design with an A-B-A-B format. Her research hypothesis 65. A forty year-old female cafeteria worker sustained a right-
states that pain rating scores will decrease with the sided injury to her back while playing golf. She was driving
treatment intervention. Acceptance of this hypothesis would the ball when she felt an immediate sharp pain in her right
be indicated if: low back. She states that in the morning she is stiff and her
A. B is equal to A. pain eases after taking a shower. Based on the above
B. B is greater than A, at the .05 level. information, the structure MOST likely involved is:
C. B is greater than A, at the 1.0 level. A. a disc.
D. B is less than A. B. a facet joint.
C. a nerve root.
A60 C2 3 D. the arterioles which supply circulation to the spinal
60. Isokinetic training can best be used in the rehabilitation of cord.
patients with stroke during the late stages of recovery to: A66 C7 3
A. improve initiation of movement. 66. At 10 a.m. a physical therapist working on a spinal cord
B. improve rate control at faster movement speeds. unit is treating a patient with paraplegia at the T3 level. The
C. improve rate control at slower movement speeds. therapist smells alcohol on his breath and the patient is
D. increase strength of synergy components. having difficulty accomplishing a bed-to-chair transfer that
was previously done without assistance. In this case the
A61 C6 3 physical therapist should:
61. A 72 year-old patient with a transfemoral amputation is A. confront the patient and ask if he has been drinking.
unable to wrap his residual limb. Your BEST course of B. document and report suspicions of alcoholism to the
action is to: rehabilitation team at the weekly meeting.
A. apply a temporary prosthesis immediately. C. document the findings and immediately inform the
B. consult with the vascular surgeon about the patient’s physician about the situation.
application of an Unna’s paste dressing. D. immediately inform the nurse in charge.
C. redouble efforts to teach proper wrapping.
D. use a shrinker. A67 C2 3
67. A patient with right hemiparesis has difficulty clearing the
affected foot during the swing phase of gait. An appropriate

’05 Sullivan A
Practice Questions 7

physical therapy intervention for the right lower extremity


might include: A73 C8 3
A. bridging. 73. The most appropriate school physical therapy intervention
B. forward step-ups in standing using graduated height to use during class for a child with decreased sitting
steps. balance, but normal tone would be to:
C. pushing backward while sitting on a rolling stool. A. adapt a desk and wheelchair to provide adequate
D. sitting on a Swiss ball, alternating lateral side steps sitting balance.
and back to neutral. B. use a prone-stander.
C. use a sidelyer.
A68 C6 2 D. use a therapeutic ball to promote sitting balance.
68. A 19 year-old patient has a complete spinal cord injury at
the level of L1. His primary goal is to walk again. You A74 C2 2
decide it would be MOST appropriate to recommend that 74. A patient with Parkinson’s disease demonstrates a highly
this patient use: stereotyped gait pattern characterized by impoverished
A. a reciprocating gait orthosis and walker. movement. The intervention that would be LEAST
B. a wheelchair, because ambulation is unrealistic. appropriate to use with this patient is:
C. bilateral AFOs and Lofstrand crutches. A. a rolling walker to compensate for impaired balance.
D. bilateral KAFOs with thoracolumbosacral extension B. postural exercises to promote trunk and head
control. extension in sitting and standing.
C. range of motion exercises to reduce hip and knee
A69 C3 2 flexion contractures.
69. A patient is four weeks post myocardial infarction. Resistive D. sidelying, upper and lower trunk rotation segmental
exercises using weights are: patterns using the technique of rhythmic initiation.
A. always contraindicated during acute and post-acute
phases of cardiac rehabilitation due to expected A75 C2 1
elevations of BP. 75. Your patient is 72 and recovering from a right CVA. She
B. appropriate if intensities are kept below 40% maximal tells you she is thirsty and asks you for a can of soda.
voluntary contraction. When you give her the can and instruct her to open it, she
C. appropriate if intensities are kept below 85% maximal is unable to complete the task. Later after the treatment
voluntary contraction. session when she is alone you observe her drinking from
D. safe during all phases of rehabilitation if judicious the can. You suspect she may have a primary deficit in:
monitoring of HR is used. A. anosognosia.
B. ideational apraxia.
A70 C5 3 C. ideomotor apraxia.
70. A patient diagnosed with cervical radiculitis has been D. unilateral neglect.
referred to you for mechanical traction. You are applying
the traction using the cervical halter for 5 minutes at 20° A76 C7 3
neck flexion using 10 pounds. The patient complains of 76. Peer review is an important professional activity. Recently,
pain in the area of the TMJ. You should: physical therapists have been the focus of vigorous peer
A. change the angle of pull. review due to increasing financial pressure imposed by
B. decrease the traction poundage. third party payers. An inappropriate use of peer review is to
C. decrease the treatment time. determine whether care:
D. discontinue the treatment. A. should be paid for by a third party payer.
B. was appropriate and required the skill of a physical
A71 C4 3 therapist.
71. On the first day following a cesarean delivery, the physical C. was cost effective.
therapist’s initial intervention would consist of teaching the D. was provided by the appropriate personnel.
new mother:
A. ankle exercises to prevent thrombophlebitis. A77 C3 1
B. assisted ambulation. 77. A 62 year-old patient has chronic obstructive pulmonary
C. assisted breathing and coughing and pelvic floor disease. The MOST likely pulmonary test result would be:
exercises. A. decreased functional residual capacity.
D. partial sit-ups and pelvic floor exercises. B. decreased residual volume.
C. increased total lung capacity.
A72 C1 2 D. increased vital capacity.
72. It is important to note the status of the pars interarticularis
on the X-ray report. A problem with this part of the vertebra A78 C6 1
could possibly lead to: 78. A patient with a transfemoral amputation is being fitted with
A. spondylolisthesis with discal herniation. a quadrilateral socket. Areas of pressure tolerance would
B. spondylolisthesis with possible anterior slippage of be expected over the:
the vertebral body. A. distolateral end of femur and ischial seat.
C. spondylolysis resulting in early nerve root B. gluteals and adductor magnus.
compression. C. ischial tuberosity, gluteals, and lateral sides of
D. spondylolysis with early degeneration of the vertebra. residual limb.
D. perineal area and medial side of femur.

‘05 Sullivan A
8 Practice Questions

A79 C4 2 D. train the teacher in manual handling techniques to


79. A patient recently diagnosed with fibromyalgia and chronic assist the child in head retraction.
fatigue immune system dysfunction demonstrates a loss of
interest in all activities and outlets. She is not eating well A83 C7 3
and is having problems sleeping. Recently she has talked 83. You have recently attended a professional conference on
about suicide as her only hope. Your BEST course of myofascial release. You wish to share this information with
action is to: your colleagues during an inservice session. You BEST
A. call her physician, saying that you cannot do anything initial activity is to:
for this patient until her psychological outlook is A. ask your colleagues about their current level of
better. knowledge using a brief questionnaire.
B. discuss her need for medications with her husband. B. determine the best sequence for the learning units
C. immediately contact her primary physician. within your presentation.
D. present a positive attitude and tell her she will feel C. provide a comprehensive packet of handouts when
better soon. you begin the first inservice session.
D. select a suitable time and place for your lecture.
A80 C6 3
80. A 77 year-old patient has been confined to bed for a period A84 C2 2
of 2 months and now demonstrates limited ROM in both 84. Following an episode of adhesive capsulitis of the right
lower extremities. Range in hip flexion is 5º to 115º and shoulder, a 52 year-old with a history of left CVA now
knee flexion is 10º to 120º. The MOST appropriate exhibits reflex sympathetic dystrophy affecting the right
intervention to improve flexibility and ready this patient for upper extremity. The intervention that should be AVOIDED
standing is: in this case is:
A. dynamic lower extremity splints, applied for 2 hours A. graduated active exercises.
daily. B. massage to reduce edema.
B. hold-relax techniques followed by passive ROM 3 C. passive manipulation to the shoulder.
times a week. D. passive ROM exercises.
C. manual passive stretching, 5 repetitions each joint, 2
times a day. A85 C6 3
D. mechanical stretching using traction and 5 lb. 85. To prepare a patient with an incomplete T12 paraplegia for
weights, 2 hours, twice daily. ambulation with crutches, the upper quadrant muscles that
would be MOST important to strengthen include the:
A81 C1 2 A. deltoid, coracobrachialis, and brachialis.
81. A seventy year-old male retired carpenter who has had B. lower trapezius, latissimus dorsi, and triceps.
long term lumbar pain with a previous diagnosis of C. middle trapezius, latissimus, dorsi and triceps.
degenerative joint disease (DJD) of his lumbar facet joints. D. upper trapezius, rhomboids, and levator scapulae.
He complains of numbness, paresthesias and weakness of A86 C3 2
his bilateral lower extremities which increase with extended 86. A 72 year-old is recovering at home from a myocardial
positions or walking greater than 100 feet. His pain persists infarction and percutaneous transluminal coronary
for hours after assuming a resting position. He reports he angioplasty. As his home care physical therapist, you
can ride his stationary bike for 30 minutes without any decide to use pulse oximetry to monitor his responses to
problems. Primary physical therapy intervention should exercise. An acceptable oxygen saturation rate (SaO2) to
include: maintain throughout the exercise period is:
A. abdominal and back extension strengthening as the A. 50%.
result of spondylolysis. B. 82%.
B. increasing cardiovascular endurance as the result of C. 85%.
degenerative arthritis. D. 92%.
C. stretching and limiting extended spinal positions as
the result of spinal stenosis. A87 C5 2
D. traction and limitation of weight bearing positions as 87. A 42 year-old homemaker presents with acute lateral
the result of discal dysfunction. epicondylitis following participation in a local tennis
tournament. Pain is 9/10. The MOST appropriate initial
A82 C7 3 intervention for this problem is:
82. A physical therapist is instructing a kindergarten teacher in A. cold intermittent compression three times per week.
a behavior management program for a child with B. cold whirlpool daily until the pain subsides.
developmental disabilities who has been mainstreamed C. fluidotherapy three times per week.
into the regular classroom. The therapist requests that the D. ice massage B.I.D. until the pain subsides.
teacher encourage the child to maintain a head retracted
sitting position in the class. The strategy that would be A88 C1 1
MOST helpful in this situation is to: 88. A patient was referred to physical therapy complaining of
A. have the teacher encourage the classmates to tell the loss of cervical AROM. His X-rays showed DJD at the
child to sit up in the chair. uncinate processes in the cervical spine. The motion that
B. have the teacher give a smile sticker when the child would be MOST restricted would be:
sits with head retracted for five minutes. A. extension.
C. have the teacher issue a verbal reprimand when the B. flexion.
child slumps in the chair. C. rotation.

’05 Sullivan A
Practice Questions 9

D. side-bending. A. dizziness or syncope.


B. dyspnea, anxiety, or disorientation.
A89 C1 2 C. muscle twitching or tetany.
89. A 55 year-old male electrician is unable to pull wire D. tingling or numbness.
overhead due to a painless inability to reach past 80
degrees of right shoulder abduction. He has had numerous A95 C1 2
previous episodes of right shoulder pain over the last ten 95. A 34 year-old female in her second trimester of pregnancy
years which were diagnosed as shoulder tendinitis. Early was sent to physical therapy with complaints of tingling and
subacute physical therapy intervention should focus on: loss of strength in both of her hands. Her symptoms are
A. active assistive pulley exercises. exacerbated if she is required to use her keyboard at work
B. gentle grade III translatory glenohumeral for longer than 20 minutes. The MOST appropriate physical
mobilizations. therapy intervention would include:
C. modalities to reduce pain and inflammation. A. dexamethasone phonophoresis to the carpal tunnel.
D. resistance exercises for the affected supraspinatus B. hydrocortisone iontophoresis to the volar surfaces of
muscle. both wrists.
C. ice packs to the carpal tunnel.
A90 C7 2 D. placing the wrists in resting splints.
90. Two therapists are asked to perform a test on the same
group of patients using the Functional Independence A96 C8 3
Measure (FIM). The results of both sets of measurements 96. A child you are working with in the elementary school
reveal differences in therapists’ scores but not in the repeat system has moderate to severe extensor spasticity and
measurements. This is indicative of a problem in: limited head control. The MOST appropriate positioning
A. concurrent validity. device would be a:
B. construct validity. A. prone stander with abduction wedge.
C. interrater reliability. B. supine stander with abduction wedge.
D. intrarater reliability. C. wheelchair with a back wedge and head supports.
D. wheelchair with adductor pommel.
A91 C2 3
91. Your patient is a 16 year-old recovering from a complete A97 C2 3
spinal cord injury with C5 quadriplegia. You are performing 97. A patient with complete C6 quadriplegia should be
PROM exercises on the mat when he complains of a instructed to initially transfer with a sliding board using:
sudden pounding headache and double vision. You notice A. pectoral muscles to stabilize elbows, scapular
he is sweating excessively, and when you take his BP it is depressors to lift trunk.
240/95. Your BEST course of action is to: B. serratus anterior to elevate trunk with shoulder
A. immediately contact the patient’s physician. extensors stabilizing.
B. lie the patient down immediately, elevate his legs, C. shoulder extensors, external rotators, and anterior
then call for a nurse. deltoid to position and lock the elbow.
C. place him in sitting position and continue to monitor D. triceps, keeping the hands flexed to protect tenodesis
BP. grasp.
D. sit the patient up, check/empty catheter, and then call
for emergency medical assistance. A98 C7 3
A92 C6 3 98. You have volunteered to teach a stroke education class on
92. A 46 year-old patient presents with a flatfoot deformity with positioning techniques for family members and caregivers.
abduction of the forefoot in relation to the weight bearing There will be 12 individuals attending this class, ranging in
line. The forefoot is inverted to the varus position when age from 42 to 82. Your BEST choice of teaching methods
inspected from the frontal plane. Corrections for this foot is to utilize:
deformity would NOT include a: A. demonstration, practice, and follow-up discussion.
A. metatarsal bar. B. lecture with some time for questions at the end of the
B. scaphoid pad. 30 minute session.
C. Thomas heel. C. multimedia (slides and overheads) to accompany
D. UCBL insert. your oral presentation.
D. one on one practice with adequate critiquing of
A93 C4 3 performance.
93. A 10-year old with full thickness burns to both arms is A99 C3 2
developing hypertrophic scars The BEST intervention to 99. A patient with active tuberculosis is referred for physical
manage these scars is: therapy. Which of the following is NOT an appropriate
A. custom made pressure garments. precaution?
B. occlusive dressings. A. have the patient wear a tight fitting mask while being
C. primary excision followed by autografts. treated in his room.
D. surgical resection (Z-plasty). B. insure that the patient is in a private, negative
pressurized room.
A94 C3 1 C. wash hands upon entering and leaving the patient’s
94. A patient with COPD has developed respiratory acidosis. room.
You instruct the physical therapy student who is D. wear a tight fitting mask while treating the patient.
participating in her care to monitor the patient closely for:

‘05 Sullivan A
10 Practice Questions

A100 C1 2 B. behind and to the left side, one hand on the gait belt.
100. A twelve-year old female figure skater has been referred to C. behind the patient with both hands on the gait belt.
your clinic for treatment for patellar tendinitis. The D. in front of the patient, walking backward, with one
examination reveals that she is unable to hop on the hand on the gait belt and one hand on the shoulder.
affected lower extremity due to pain. You decide to refer E. A105 C5 1
her back to her pediatrician suggesting that the patient 105. A 13 year-old severed the median nerve three days ago
receive an x-ray of her knee. The patient returns for when his hand went through a glass window. To determine
therapy with the x-ray shown in the figure. Your initial the motor function of the nerve you perform a chronaxie
intervention should focus on: Magee D (2002). Orthopedic test. At this time you would expect the chronaxie of the
Physical Assessment, 4th ed. Philadelphia, W. B. nerve to be:
Saunders, Figure 12-149, page 746, with permission. A. absent.
A. aggressive plyometric exercises with focus on B. decreased.
endurance training. C. increased.
B. fitting patient with crutches for non weight-bearing D. unaffected.
ambulation and initiation of phonophoresis at 3mHz
continuous wave using hydrocortisone. A106 C1 3
C. patient education regarding avoidance of squatting 106. A patient has lumbar spinal stenosis encroaching on the
and jumping activities as well as initiation of spinal cord. The physical therapist should educate the
iontophoresis using dexamethasone. patient to AVOID:
D. patient education regarding avoiding falls onto her A. bicycling.
affected knee and open chain knee extension B. rowing.
exercises to improve quadriceps strength. C. swimming using a crawl stroke.
D. Tai Chi.
A101 C6 2
101. A 40 year-old male with a history of low back pain has been A107 C6 2
receiving physical therapy for 12 weeks. The patient is 107. A patient with post-polio syndrome presents in your clinic
employed as a loading dockworker. He performs repetitive with symptoms of myalgia and increasing fatigue. He is
lifting and carrying of boxes weighing between 15 and 30 wearing a KAFO which he has had for 10 years. When
pounds. An appropriate engineering control to reduce the walking, you observe that he rises up over the sound limb
stresses of lifting and carrying would be to: to advance the orthotic limb forward. Your BEST
A. issue the employee a back support belt. intervention is to provide:
B. provide a two-wheel handcart for use in moving the A. a manual wheelchair with reclining back and elevating
boxes. legrests.
C. require the worker to attend a class in using correct B. a shoe lift on the orthotic side.
body mechanics while performing the job. C. a shoe lift on the sound side.
D. use job rotation. D. an electric wheelchair with joystick.

A102 C1 1 A108 C5 2
102. Common compensatory postures you would expect for a 108. Your patient has moderate spasticity of the biceps brachii
patient diagnosed with fixed severe forefoot varus are: on the left as a result of a CVA. You choose to use
A. excessive ankle dorsiflexion and medial rotation of electrical stimulation to temporarily decrease the effects of
the femur. hypertonicity in order to work on ADL activities. Your
B. excessive midtarsal supination and lateral rotation of objective in applying the current is to:
the tibia. A. fatigue the ipsilateral biceps brachii.
C. subtalar pronation and medial rotation of the tibia. B. stimulate the contralateral biceps brachii.
D. toeing-in and lateral rotation of the femur. C. stimulate the contralateral triceps.
D. stimulate the ipsilateral triceps.
A103 C1 2
103. A patient has undergone surgery and subsequent A109 C8 2
immobilization to stabilize the olecranon process. The 109. You are working with a four year-old child who has
patient now exhibits an elbow flexion contracture. In this myelodysplasia at the L5 level. At this level the most
case, an absolute CONTRAINDICATION for joint appropriate orthosis to recommend for ambulation would
mobilization would be: be a (an):
A. empty end-feel. A. ankle-foot orthosis.
B. firm end-feel. B. knee-ankle-foot orthosis.
C. soft end-feel. C. parapodium.
D. springy end-feel. D. reciprocating gait orthosis.

A104 C6 3 A110 C1 2
104. You observe a physical therapist assistant ambulate a 110. An eleven-year-old male was referred to physical therapy
patient for the first time after a left total hip replacement. with complaints of vague pain at his right hip and thigh
The patient is using crutches and is practicing on a level which radiates to his knee. His AROM is restricted in
surface. The PTA should guard the patient by standing abduction, flexion, and internal rotation. A gluteus medius
slightly: gait was observed with ambulation for 100 feet. Appropriate
A. behind and to the intact side, one hand on the gait PT intervention would include:
belt.

’05 Sullivan A
Practice Questions 11

A. closed-chain partial weight-bearing lower extremity C. supine with the affected hand positioned on stomach.
exercises for slipped capital femoral epiphysis. D. supine with the affected upper extremity positioned
B. hip joint mobilization to improve the restriction in close to the side of the trunk.
motion as the result of Legg-Calvé Perthe’s disease.
C. open-chain strengthening of his right hip abductors A115 C3 2
and internal rotators for avascular necrosis of the hip. 115. A patient presents with a large plantar ulcer that will be
D. orthoses to control lower extremity position as the debrided in the whirlpool. The foot is cold, pale, and
result of femoral anteversion. painless. The condition that would most likely result in this
A111 C7 2 clinical presentation is:
111. A researcher states that he expects that there will be no A. acute arterial insufficiency.
significant difference between 20 and 30 year-olds after a B. chronic arterial insufficiency.
12 week exercise training program using exercise heart C. chronic venous insufficiency.
rates and myocardial oxygen consumption as measures of D. deep venous thrombosis.
performance. The kind of hypothesis that is being used in
this study is a (an): A116 C5 3
A. directional hypothesis. 116. A patient has been referred to you following a fracture of
B. experimental hypothesis. the femur six months ago. The cast was removed, but the
C. null hypothesis. patient was unable to volitionally contract the quadriceps.
D. research hypothesis. You decide to apply electrical stimulation to the quadriceps
muscle. Your choice of electrode placement and electrical
A112 C7 3 stimulation duty cycle (on:off ratio) would consist of:
112. A physical therapist was treating a patient and the patient A. large electrodes, closely spaced; 10:30.
in the next bed was uncomfortable and asked the therapist B. large electrodes, widely spaced; 10:30.
to move his leg. The therapist placed the leg on 2 pillows C. small electrodes, closely spaced; 10:30.
as requested by the patient. Unknown to the therapist this D. small electrodes, widely spaced; 10:10.
patient had a femoral artery graft 2 days previously. As a
result the graft became occluded and the patient was A117 C4 2
rushed to surgery for a replacement. The patient claimed 117. While setting a patient up for cervical traction, you notice a
the therapist placed his leg too high on the pillows causing purplish mole with rough edges on the patient’s neck. You:
the occlusion of the original graft and sued for malpractice. A. call the physician immediately and report your
The hospital administrator decided: findings.
A. it was the patient’s fault for requesting the position B. document the skin condition and keep a watchful eye
change and therefore supported the action of the on it.
physical therapist. C. tell the patient if it bleeds at all to report it to his
B. that the physical therapist was functioning according physician.
to common protocols of the institution and thus D. treat the patient but cover the mole with a gauze pad.
supported the actions of the therapist.
C. that the therapist was functioning outside the
A118 C2 1
common protocols of the hospital, and therefore did
118. A 15 year-old male suffered traumatic brain injury and
not support the actions of the physical therapist.
multiple fractures following a motor vehicle accident. He is
D. to counter-sue the patient because he was
recovering in the intensive care unit. Your referral states
responsible for requesting the position change.
PROM and positioning. On day 1 he is semi-alert and drifts
in and out while you are working with him. On day 2 you
A113 C2 3 become concerned because you observe signs suggestive
113. A 26 year-old female has a 3 year history of multiple of increasing intracranial pressure. You promptly report
sclerosis. One of her disabling symptoms is a persistent these symptoms to his physician. The signs that would be
and severe diplopia which leaves her frequently nauseated cause for immediate action in this case would be:
and immobile. An appropriate intervention strategy to assist A. decreasing consciousness with slowing of pulse and
her in successfully participating in rehabilitation would be Cheyne-Stokes respirations.
to: B. decreasing function of cranial nerves IV, VI, and VII.
A. give her a soft neck collar to limit head and neck C. developing irritability with increasing symptoms of
movements. photophobia, disorientation and restlessness.
B. give her special glasses which magnify images. D. positive Kernig’s sign with developing nuchal rigidity.
C. have her close her eyes and practice movements
without visual guidance.
A119 C2 1
D. patch one eye.
119. You have received a referral for a 42 year-old patient who
has a neurapraxia involving the ulnar nerve secondary to
A114 C2 3 an elbow fracture. Based on your knowledge of this
114. The MOST appropriate positioning strategy for a patient condition, you expect that:
recovering from acute stroke who is in bed and A. regeneration is unlikely because surgical
demonstrates a flaccid upper extremity is: approximation of the nerve ends was not performed.
A. sidelying on the affected side with the affected upper B. nerve dysfunction will be rapidly reversed, generally
extremity flexed overhead. in 2-3 weeks.
B. sidelying on the sound side with the affected upper C. regeneration is likely after 2-21/2 years.
extremity supported on a pillow with the shoulder D. regeneration is likely in 6-8 months.
protracted and elbow extended.

‘05 Sullivan A
12 Practice Questions

A120 C7 3 spite of increasing the intensity to maximum . You should


120. After three weeks of teaching a patient how to ambulate now advise the patient to:
with bilateral crutches and a touch down gait, you A. decrease the pulse duration.
determine the most appropriate kind of feedback to give to B. increase the treatment frequency.
the patient is: C. switch to low rate TENS.
A. continuous feedback in which you provide ongoing D. switch to modulation mode TENS.
verbal cuing during gait.
B. immediate feedback given after each practice trial. A126 C2 1
C. intermittent feedback given at scheduled intervals, 126. During initial standing a patient is pushing backward
every other practice trial. displacing the center-of-mass at or near the posterior limits
D. occasional feedback given when consistent errors of stability. The most likely cause of this is contraction of
appear. the:
A. gastrocnemius-soleus.
A121 C8 2 B. hamstrings.
121. You are a home health physical therapist. During one of C. hip extensors.
your regularly scheduled visits with a 72 year-old male D. tibialis anterior/peroneals.
patient, you find him to be confused with shortness of
breath and significant generalized weakness. Assessing A127 C7 2
these symptoms and given his history of hypertension and 127. A patient has been screened using a new test for the
hyperlipidemia, you suspect: presence of a gene (ALG-2) linked to Alzheimer’s disease.
A. he forgot to take his hypertension medication. His physician reports he lacks the gene and should not be
B. he may be experiencing unstable angina. at increased risk to develop the disease. Some years later
C. he may be presenting with early signs of myocardial he develops Alzheimer’s and a repeat test reveals the
infarction. presence of the gene. The results of the initial test can be
D. his mental changes are indicative of early Alzheimer’s interpreted as:
disease. A. false negative.
B. false positive.
A122 C1 2 C. high degree of sensitivity.
122. Physical therapy intervention for a sixty-five year-old male D. high degree of specificity.
patient with a recent diagnosis of supraspinatus tendinitis
with possible impingement syndrome of the right shoulder A128 C8 1
should emphasize: 128. A 62 year-old lives at home with his wife and adult
A. joint mobilization, use of ice, and rotator cuff daughter. He has recently been diagnosed with multi-infarct
strengthening. dementia and is recovering from a fractured hip following a
B. modalities to reduce inflammation, active assistive fall injury. In your initial interview with his wife you would
range of motion exercises using pulleys, and postural expect to find:
realignment. A. agitation and sundowning.
C. reducing stresses to abnormal tissues by placing the B. history of steady progression of loss of judgment and
right upper extremity in a sling, use of ice, and rotator poor safety awareness.
cuff strengthening. C. history of sudden onset of new cognitive problems
D. rest to reduce pain, iontophoresis, and strengthening and patchy distribution of deficits.
of the rotator cuff muscles. D. perseveration on a thought or activity.
A123 C3 3 A129 C1 2
123. The recommended time duration for endotracheal 129. A fourteen-year-old girl complains of subpatellar pain after
suctioning is: participation in an aerobic exercise program for two weeks.
A. 1 to 5 seconds. Your examination shows a large Q angle, pain with
B. 10 to 15 seconds. palpation at the inferior pole of the patella, and mild
C. 15 to 20 seconds. swelling at both knees. Physical therapy intervention
D. 5 to 10 seconds. should promote:
A. hamstring strengthening.
A124 C3 3 B. lateral patellar tracking.
124. A 76 year-old frail older adult is confined to bed in a nursing C. vastus lateralis strengthening.
facility. He has developed a small superficial wound over D. vastus medialis muscle strengthening.
the sacral area. Since only small amounts of necrotic tissue
are present, the physician has decided to use autolytic A130 C2 3
wound debridement. This is BEST achieved with: 130. You receive a referral from an acute care physical therapist
A. forceful irrigations. to treat a patient with right hemiparesis in the home. The
B. hydrotherapy. referral indicates that the patient demonstrates good
C. occlusive dressings. recovery: both involved limbs are categorized as stage 4
D. wet-to-dry dressings. (Brunnstrom recovery stages). He is ambulatory with an
small-based quad cane. The activity that would be MOST
A125 C5 3 appropriate for a patient at this stage of recovery is:
125. Three weeks ago, you instructed your patient in applying A. sitting, marching in place (alternate hip flexion
conventional (high rate) TENS to the low back to modulate movements).
a chronic pain condition. The patient now states that the B. standing, picking the foot up behind and slowly
TENS unit is no longer effective in reducing the pain in lowering it.

’05 Sullivan A
Practice Questions 13

C. standing, small range knee extension to gain A136 C1 3


quadriceps control. 136. In treating a patient with a diagnosis of right shoulder
D. supine, bending the hip and knee up to the chest with impingement syndrome, the therapist should not allow the
some hip abduction. patient to perform assisted repetitive overhead exercises
without FIRST:
A131 C4 2 A. controlling all pain.
131. During a physical therapy session for low back pain, a 67 B. having complete AROM at the shoulder.
year-old patient tells you that she has had urinary C. instruction in proper postural alignment.
incontinence for the last year. It is particularly problematic D. stretching the shoulder girdle muscles.
when she has a cold and coughs a lot. She has not told her
doctor about this problem because she is too embarrassed. A137 C8 3
Your BEST course of action is to: 137. An older person with a diagnosis of degenerative joint
A. examine the patient and proceed with her back disease should understand his disease, its progression,
treatment. and its management. You will know your patient education
B. examine the patient, document and discuss your has been effective if he can tell you:
findings with the doctor. A. aerobic conditioning is not appropriate when pain is
C. examine the patient, document the problems, then present and medications must be used.
send her back to her doctor. B. joint protection strategies are important but cannot
D. refer the patient back to her doctor. reduce the expected loss of function.
C. loss of ROM and immobility are expected and
A132 C2 1 irreversible.
132. During a sensory exam you determine that a patient D. pain and stiffness are worse in the early morning and
complains of a dull, aching pain but is not able to should decrease with moderate activity.
discriminate a stimulus as sharp or dull. Two-point
discrimination is absent. Based on these findings, the A138 C2 1
pathway that is intact is the: 138. A 34 year-old patient is recently admitted to your facility
A. anterior spinothalamic tract. with a diagnosis of Guillain-Barré syndrome. In your initial
B. dorsal columns/neospinothalamic systems. examination, you expect to find:
C. fasciculus gracilis/medial lemniscus. A. asymmetrical weakness with hyperreflexia, bulbar
D. lateral spinothalamic tract. palsy.
B. glossopharyngeal and vagal paralysis with
A133 C3 2 hyperactive jaw and snout reflexes.
133. A patient is five days post-MI and is receiving cardiac C. sensory loss (stocking and glove distribution) with
rehabilitation. At this time a goal that would be minor loss of motor function.
INAPPROPRIATE is to: D. symmetrical distribution of weakness, ascending with
A. counteract deconditioning associated with bed rest. possible involvement of lower cranial nerves.
B. educate the patient and family regarding risk factor
reduction. A139 C5 3
C. increase the patient’s maximal oxygen consumption 139. A 31 year-old patient presents with patellar tendinitis as a
by discharge. result of a mountain climbing accident three weeks ago.
D. initiate early return to independence in activities of The patient complains of pain on resisted knee extension,
daily living. stair climbing, and sit-to-standing movements. You choose
to apply iontophoresis using a pain medication with a
A134 C6 3 positive charge. The correct current type, polarity and
134. A 73 year-old man has bilateral short transfemoral (AK) active electrode placement is:
amputations and will require a wheelchair for functional A. biphasic current with the cathode placed proximal on
mobility in the home and community. An appropriate the tendon.
prescription for his wheelchair includes: B. low volt continuous current with the anode placed
A. increasing the seat depth by 2 inches to distal on the tendon.
accommodate the length of the residual limbs. C. monophasic current with the anode placed on the
B. lowering the seat height by 3 inches. tendon.
C. placement of the drive wheels 2 inches anterior to the D. monophasic current with the cathode placed on the
vertical back supports. tendon.
D. placement of the drive wheels 2 inches posterior to
the vertical back supports. A140 C6 3
A135 C7 2 140. A 52 year-old patient sustained a T10 spinal cord injury
135. A multicenter study was done on the reliability of passive four years ago. During initial examination you observe
wrist flexion and extension goniometric measurements redness over the ischial seat that persists for 10 minutes
using volar/dorsal alignment, ulnar alignment and radial when not sitting. The BEST intervention in this case would
alignment. Significant differences were revealed between be to:
the three techniques. An appropriate level for determining A. change to a low density wheelchair cushion.
significant difference is a P value of: B. have the patient do sitting push-ups at least every 10
A. P=0.015 minutes.
B. P=0.05 C. increase the arm rest height.
C. P=0.1 D. order a tilt-in-space wheelchair.
D. P=0.5

‘05 Sullivan A
14 Practice Questions

A141 C1 1 A146 C4 2
141. A 16 year-old adolescent female was sent to physical 146. A 24 year-old woman recently delivered twins on the
therapy with a diagnosis of anterior knee pain. Positive obstetrical service of your hospital. After delivery she has
findings include pes planus, lateral tibial torsion and genu developed a 4 centimeter diastasis recti abdominis. The
valgum. The position the femur will be in is excessive: BEST initial intervention for this problem is to teach:
A. abduction. A. gentle stretching of hamstrings and hip flexors.
B. lateral rotation. B. pelvic floor exercises and sit-ups.
C. medial rotation. C. pelvic tilts and bilateral straight leg raising.
D. retroversion. D. protection and splinting of the abdominal
musculature.
A142 C2 3 A147 C3 3
142. A 24 year-old patient in a coma recovering from traumatic 147. The optimal position for ventilation of a patient with a C5
brain injury is receiving PROM exercises. The family is complete spinal cord injury is:
confused because the occupational therapist has told them A. semi Fowler’s.
to do range of motion exercises in a different way than the B. sidelying, head of bed elevated 45 degrees.
physical therapist had instructed. As the physical therapist C. sidelying, head of bed flat.
it would be BEST to deal with this situation by telling the D. supine, head of bed flat.
family:
A. the PT and OT supervisors will review and correct A148 C7 2
any discrepancies in the exercise program. 148. A 16 year-old patient with osteosarcoma is being seen in
B. there is no need for them to worry and note the physical therapy for crutch training. Her parents have
problem in the patient’s record. decided not to tell her about her diagnosis. She is quite
C. to exercise the way you instructed until any perceptive and asks you directly if she has cancer and
differences can be worked out with the occupational about her future. Your BEST course of action is to:
therapist. A. change the subject and discuss the plans for that
D. you will meet with the occupational therapist to day’s treatment.
discuss the exercise approaches and you will let them B. discuss her condition gently indicating her parent’s
know the outcome of the meeting as soon as fears about not telling her the diagnosis.
possible. C. schedule a conference with the doctor and family
about her condition and your discussions with the
A143 C7 3 patient.
143. You are working with a 10 year-old girl with cerebral palsy. D. tell the patient that you don’t know the specifics of her
Part of the exercises in her plan of care involve using the condition or prognosis, and she should speak with her
Swiss ball. The choice of educational media that is BEST to doctor.
use when instructing her in use of this device is:
A. a oral presentation that uses transparencies of Swiss A149 C2 2
ball positions. 149. Your patient has had amyotrophic lateral sclerosis for the
B. a slide presentation of exercises using the Swiss ball. past two years with mild functional deficits. He is still
C. a videotape of another child with cerebral palsy on a ambulatory with bilateral canes but is limited in his
Swiss ball. endurance. An important goal for his physical therapy plan
D. printed handouts with stick figure drawings and of care should be to prevent:
instructions. A. further gait deterioration as a result of ataxia.
B. myalgia.
A144 C3 3 C. overwork damage in weakened, denervated muscle.
144. As a physical therapist you are the health professional in D. radicular pain and paresthesias.
charge during a high school football game. During the
game, a player is tackled violently by two opponents. You A150 C6 3
determine that the player is unresponsive. Your immediate 150. A 28 year-old patient has extensive full thickness burns to
course of action should be to: the dorsum of the hand and forearm. He is to be fitted with
A. ask for help to log roll the player on his back while a resting splint to support his wrists and hands in a
stabilizing his neck. functional position. An appropriately constructed splint
B. open the airway by using the chin-lift method. positions the wrist and hand in:
C. stabilize the neck and flip back the helmet face mask. A. neutral wrist position with IP extension and thumb
D. summon emergency medical services. flexion.
B. neutral wrist position with slight finger flexion and
A145 C8 2 thumb flexion.
145. During a home visit, the mother of an 18 month-old child C. slight wrist extension with fingers supported and
with developmental delay and an atrio-ventricular shunt for thumb in partial opposition and abduction.
hydrocephalus tells you that her daughter vomited several D. slight wrist flexion with IP extension and thumb
times, was irritable and is now lethargic. Your BEST course opposition.
of action is to:
A. call the girl’s pediatrician immediately. A151 C3 1
B. give the child a cold bath to try and rouse her. 151. Your patient is 82 years-old with a long history of
C. give the child clear liquids since she vomited. congestive heart failure. You are alert to the signs of left-
D. place the child in a sidelying position and monitor vital sided heart failure associated with activity. The MOST
signs. important indicators include:

’05 Sullivan A
Practice Questions 15

A. bilateral ankle swelling within 2-3 hours post exercise. 157. A 24 year-old pregnant woman who is 12 weeks pregnant
B. complaints of fatigue with increasing dyspnea and asks you if it is safe to continue with her aerobic exercise.
cough. Currently she jogs 3 miles, 3 times a week. Your BEST
C. nausea and anorexia. answer is:
D. sudden weight gain within 24 hours following A. continue jogging only until the 5th month of
exercise. pregnancy.
B. during the last trimester do not jog but switch to
A152 C3 2 exercising in the supine position only.
152. A patient using an incentive spirometer complains of feeling C. jogging is safe but you might want to switch to
lightheaded. Your instructions to the patient should be to: swimming during later months.
A. lie down while using the spirometer. D. jogging is safe but your target HR should not exceed
B. take a deeper breath on the following attempt. 140 beats/min.
C. take a rest period and only use the device 10 times
per hour. A158 C6 3
D. try to use the spirometer more frequently to get used 158. Your patient returns to P.T. after his first exercise session
to it. complaining of muscle soreness that developed later in the
A153 C8 3 evening after his first session and continued into the next
153. An eighteen month-old child with Down Syndrome and day. He is unsure he wants to continue with exercise. You
moderate developmental delay is being treated at an Early can minimize the possibility of this happening again by
Intervention Program. The schedule that would best using:
facilitate motor learning of this child is physical therapy A. concentric exercises, 3 sets of 10, at 80% of maximal
intervention given: intensity.
A. 30 minutes each day. B. concentric exercises, 3 sets of 10, with gradually
B. once a week for two hours. increasing intensity.
C. one hour each week with a portion of that time used C. eccentric exercises, 1 set of 10, lifting body weight
to teach the caretaker a home program to be done 3 (sit-to-stand).
times a week. D. eccentric exercises, 3 sets of 10, with gradually
D. three times a week for fifteen minutes. increasing intensity.
159 C1 1
A154 C1 1 159. Upon examining a patient with vague hip pain which
154. A patient has been referred to you for acute shoulder pain radiates to the lateral knee, you have found a negative
after shoveling snow in a driveway for two hours. Positive FABERE test, negative grind test, and a positive Noble
findings include pain and weakness with flexion of an compression test. The dysfunction is most likely due to:
extended upper extremity as well as scapular winging with A. a possible fracture of the femoral neck.
greater than 90 degrees of abduction. The patient’s B. an iliotibial band friction disorder.
problem is MOST LIKELY the result of: C. degenerative joint disease of the hip.
A. compression of the long thoracic nerve. D. sacroiliac joint dysfunction.
B. compression of the suprascapular nerve.
C. subdeltoid bursitis. A160 C2 1
D. supraspinatus tendinitis. 160. A 22 year-old male suffered carbon monoxide poisoning
from a work-related factory accident. He is left with
A155 C2 1 permanent damage to his nervous system, affecting the
155. A patient with multiple sclerosis demonstrates strong basal ganglia. You logically expect his symptoms to
bilateral lower extremity extensor spasticity in the typical include:
distribution of antigravity muscles. You would expect this A. impaired sensory organization of balance.
patient to demonstrate: B. motor paralysis.
A. sacral sitting. C. muscular spasms and hyperreflexia.
B. sitting with both legs abducted and externally rotated. D. problems with motor planning and scaling of
C. sitting with the pelvis tilted, weight bearing on ischial movements.
tuberosities.
D. skin breakdown on the ischial tuberosities and lateral A161 C2 1
malleoli. 161. A patient presents with problems with swallowing. When
you test for phonation by having the patient say “AH” with
A156 C4 1 his mouth open, you notice there is deviation of the uvula
156. A 58 year-old woman with osteopenia has been on to one side. You then test for function of the gag reflex and
Premarin for the past 4 years. Based on your knowledge of notice loss of response to stimulation. These findings
estrogen replacement therapy, you recognize this patient is suggest involvement of the:
more susceptible to: A. facial nerve.
A. fractures. B. hypoglossal nerve.
B. osteoporosis. C. trigeminal nerve.
C. peripheral edema. D. vagus nerve.
D. vasomotor symptoms (hot flashes).
A162 C3 1
A157 C4 2 162. A patient with COPD is sitting in a bedside chair. The
apices of the lungs in this position compared with other
areas of the lungs in this position would demonstrate:

‘05 Sullivan A
16 Practice Questions

A. increased perfusion. B. excessive extensor synergy.


B. increased volume of air at resting end expiratory C. excessive flexor synergy.
pressure (REEP). D. inadequate contraction of the ankle dorsiflexors.
C. the highest changes in ventilation during the
respiratory cycle. A168 C7 3
D. the lowest oxygenation and highest CO2 in blood 168. A patient’s daughter wants to look at her father’s medical
exiting this zone. record. He has recently been admitted for an insidious
onset of low back pain. You, as the physical therapist,
A163 C6 1 should:
163. Checkout for a lower limb orthosis includes inspection of A. give her the chart and let her read it.
the alignment of anatomic and orthotic joints. During a B. tell her she cannot see the chart because she could
sagittal plane check-out you determine that the orthotic hip misinterpret the information.
joint is malaligned. The correct position is: C. tell her that she must have the permission of her
A. 3 inches below the anterior superior iliac spine. father before she can look at the chart.
B. just anterior and superior to the greater trochanter. D. tell her to ask the physician for permission
C. just posterior and inferior to the greater trochanter.
D. lateral to the greater trochanter. A169 C5 2
169. A 48 year-old female has had a total knee replacement.
A164 C7 3 Following surgery, you place her on a regimen of
164. A physical therapist assistant you supervise treated a continuous passive motion. The primary purpose for
patient in the home care setting. The patient is status post applying CPM for the first few hours is to:
CVA. Part of the plan of care includes “progressive gait A. decrease edema.
training on level surfaces”. The patient falls and sustains a B. decrease pain.
fractured hip during a visit done by the PTA. The fall C. increase range of motion.
occurred when the PTA took the patient on the stairs for the D. increase tissue tensile strength.
first time. The responsible party in this case is:
A. both the PT and the PTA because the PT gave A170 C5 2
inadequate supervision, and the PTA used poor 170. A 37 year-old male developed acute bicipital tendinitis two
judgment. days after water skiing. He is unable to work as a result of
B. neither the PT nor the PTA because patients who the pain he rates as 8/10. He is referred to physical therapy
have sustained a CVA are always at high risk for for iontophoresis treatment to help relieve the pain. For the
falling, and thus it is a regrettable occurrence only. first two weeks this patient should optimally be seen:
C. the PT who is negligent for failing to provide adequate A. five times per week
supervision of the PTA. B. once a week.
D. the PTA who is completely liable because the plan of C. three times per week.
care was altered without communicating with the D. two times per week
supervising PT.
A171 C6 1
171. You observe genu recurvatum as a patient with hemiplegia
A165 C1 3 ambulates using his posterior leaf spring (PLS) orthosis. A
165. As the result of blunt trauma to the quadriceps femoris factor that is NOT a possible cause of this gait deviation is:
muscle, a patient experiences loss of knee function. Early A. extensor spasticity.
PT interventions should stress: B. hip flexion contracture.
A. aggressive open-chain strengthening of the C. pes equinus.
quadriceps femoris to regain normal lower extremity D. quadriceps weakness.
strength. A172 C3 2
B. aggressive soft tissue stretching to remove blood 172. During a home visit you are providing postural drainage in
which has accumulated in soft tissues. the Trendelenburg position to a 15 year-old male with
C. gentle AROM exercises in weight bearing. cystic fibrosis. The patient suddenly complains of right-
D. gentle PROM exercises in nonweightbearing to sided chest pain and shortness of breath. On auscultation,
regain normal knee motion. there are no breath sounds on the right. The physical
therapist should:
A166 C8 3 A. call emergency medical technicians as it may be a
166. An important adjunct to physical therapy management of a pneumothorax.
child with moderate spastic hemiplegia would be use of: B. continue treating as it is possibly a mucous plug.
A. a KAFO on the affected side. C. place the right lung in a gravity dependent position to
B. a posterior walker. improve perfusion.
C. a tone inhibiting ankle-foot orthosis (AFO). D. reposition patient in the head of bed flat position as
D. an anterior rollator walker. Trendelenburg is causing shortness of breath.

A167 C2 1 A173 C2 3
167. While evaluating the gait of a patient with right hemiplegia, 173. A 65 year-old patient with multiple sclerosis is being treated
you note foot drop during midswing on the right. The MOST at home. The patient is bedridden for most of the day with
LIKELY cause of this deviation is: only short periods up in a bedside chair. Medicare is
A. decreased proprioception. funding the patient’s home care program which has as its

’05 Sullivan A
Practice Questions 17

primary goals maintaining PROM and positioning to D. strengthening of anterior chest muscles, costal
prevent deformity. Your role as the physical therapist is to expansion, and stretching of scapular stabilizers.
provide: A178 C2 1
A. a limited cardiovascular conditioning (sitting) program 178. A 72 year-old male was referred for rehabilitation following
aimed at improving respiratory capacity. a middle cerebral artery stroke. Based on this diagnosis
B. a restorative exercise program aimed at improving you suspect he will present with:
upright sitting control and improved functional A. aphasia since his right hemisphere is involved and he
independence. is right hand dominant.
C. PROM exercises 2 times a day with additional family B. contralateral hemiparesis and sensory deficits with
instruction to ensure weekend coverage. greater involvement in the arm than the leg.
D. supervision of home health aides for completion of a C. contralateral hemiparesis and sensory deficits with
daily home exercise program. greater involvement of leg than the arm.
D. homonymous hemianopsia, contralateral hemiplegia
A174 C2 3 with thalamic sensory syndrome.
174. Your patient is a 72 year-old man with an 8 year history of
Parkinson’s disease. He demonstrates significant rigidity, A179 C3 1
decreased PROM in both upper extremities in the typical 179. The cardiac rehabilitation team is conducting education
distribution, and frequent episodes of akinesia. The classes for a group of patients. The focus is on risk factor
exercise that BEST deals with these problems would be: reduction and successful life style modification. A
A. modified plantigrade, isometric holding, stressing participant asks you which is the bad cholesterol that
upper extremity shoulder flexion. increases atherosclerosis and risk of heart disease. Your
B. PNF bilateral symmetrical upper extremity D2 flexion answer is:
patterns, rhythmic initiation. A. all cholesterol is bad and total levels should be below
C. quadruped position, upper extremity PNF D2 flexion 200 mg/dl.
and extension. B. cholesterol is not the important factor; the
D. resistance training, free weights for shoulder flexors triglycerides should be kept low.
at 80% of 1 repetition max. C. elevated levels of high-density lipoprotein or HDL.
D. elevated levels of low-density lipoprotein or LDL.
A175 C7 3
175. A 72 year-old patient with diabetes is recovering from A180 C7 3
recent surgery to graft a large decubitus ulcer over the heel 180. An attractive physical therapist is treating a young and
of her left foot. You are concerned that her loss of range at handsome football player with an ACL sprain. She is very
the ankle (-5° to neutral) will limit her ambulation and fond of this patient and enjoys treating him. After a few
independent status. One afternoon you are very busy and visits, the football player asks her out to dinner. The
request that one of the physical therapy aides do her range physical therapist’s response should be to:
of motion exercises. The aide is new to your department A. thank him very much, and accept his offer for dinner.
but tells you she is willing to take this challenge on if you B. thank him very much, and invite him for dinner at her
show her how to do it. Your BEST course of action is to: apartment with other guests.
A. modify the treatment session to eliminate the ROM C. thank him very much, but refuse his invitation while
exercises and have the aide walk the patient in the he is receiving treatment.
parallel bars. D. transfer the patient care to one of her colleagues and
B. perform the ROM exercises yourself. then go out to dinner with him.
C. reschedule the patient for tomorrow when you have
more time. A181 C1 2
D. take 5 minutes to instruct the aide in ROM exercises, 181. After running one mile, an athlete complains of deep
then have her do them. cramping at the anterior aspects of the legs which does not
ease and prevents continued running. Management of this
A176 C8 2 problem should include:
176. The MOST appropriate type of adaptive equipment for a A. orthotic fabrication to enable continued running on all
teenager with severe athetosis and severe extensor surfaces
spasms of the lower extremities would be: B. referral to a physician to evaluate anterior
A. posterior rollator walker. compartment pressures during activity.
B. scooter board. C. referral to a physician to rule out spinal stenosis.
C. supine stander with head support. D. stretching of the tibialis anterior muscles to help
D. wheelchair with head and trunk support with abductor resolve shin splints.
wedge.
A182 C2 2
A177 C1 2 182. A 62 year-old woman developed polio at the age of 6 with
177. The symptoms of ankylosing spondylitis in its early stages significant lower extremity paralysis. She wore bilateral
can best be managed in physical therapy by: long leg braces for a period of 2 years. She then recovered
A. back extension, costal expansion exercises, and enough to stop using her braces but still required bilateral
maintenance of proper posture to prevent deformity. Lofstrand crutches, then bilateral canes to ambulate.
B. cardiovascular conditioning, weight bearing Recently she has been complaining of new difficulties (she
exercises, and joint protection education. has had to start using her crutches again). You suspect
C. pain management, abdominal strengthening, and post-polio syndrome. The BEST goal/intervention for this
breathing exercises. patient based on her current findings is to:

‘05 Sullivan A
18 Practice Questions

A. prescribe an aquatic therapy program consisting of 188. A 72-year old patient is receiving outpatient physical
daily 1 hour aerobics. therapy at your private clinic. The clinic is an approved
B. put her on a lower extremity resistance training Medicare outpatient provider. Your patient is concerned
program utilizing 80% one repetition max. that she will not be able to pay for her continuing care and
C. teach her activity pacing and energy conservation worries that her Medicare benefits will run out soon. You
techniques. tell her:
D. utilize a moderate conditioning program consisting of A. coverage is limited only for hospital-based outpatient
cycle ergometry 3 times a week of 60 minutes at 60% PT services.
maximal heart rate. B. currently there is no limit to her Medicare coverage
for outpatient PT services.
A183 C1 1 C. there is currently a limit of $1000.00 for coverage of
183. A patient diagnosed with lumbar spondylosis without discal outpatient PT services.
herniation or bulging has a left L5 neural compression. The D. there is currently a limit of $1590.00 for coverage of
most likely structure compressing the nerve root is: outpatient PT services.
A. anterior longitudinal ligament.
B. ligamentum flavum. A189 C2 3
C. posterior longitudinal ligament. 189. Your patient is a 42 year-old woman who suffered a stroke
D. supraspinous ligament. and demonstrates a locked-in state characterized by
A184 C3 2 spastic quadriplegia and bulbar palsy. To facilitate
184. During an exercise tolerance test (ETT) a patient communication with this patient you should instruct the
demonstrates poor reaction to increasing exercise intensity. family to:
According to the American College of Sports Medicine, an A. give her a chance to mouth her responses even
absolute indication for terminating this test is: though she can’t vocalize well.
A. 1.5 mm of downsloping ST-segment depression. B. look closely at her facial expression to detect signs of
B. fatigue and shortness of breath. what she is trying to communicate.
C. onset of moderate to severe angina. C. use a communication board with minimal movements
D. supraventricular tachycardia. of her hand.
D. use an alternate eyelid taping schedule which will
A185 C2 3 allow her to use eye movements to communicate.
185. A patient with T10 paraplegia is receiving daily ROM
exercises. On this day you notice swelling, pain, local A190 C6 2
warmth and erythema in the thigh near the hip joint with 190. After performing an ergonomic examination of a worker
some limitation in motion. Your BEST course of action is: and workstation the most appropriate recommendation for
A. apply ice before his ROM exercises to ease his pain achieving ideal wrist and elbow positioning would be to:
and discomfort and inflammation. A. add armrests.
B. document the findings in his record and double his B. elevate the keyboard to increase wrist flexion.
daily sessions of ROM exercises. C. lower the keyboard to increase wrist extension.
C. instruct his aides to be more vigorous in positioning D. maintain the keyboard in a position allowing a neutral
and ROM exercises. wrist position.
D. notify the physician immediately, you suspect
heterotopic ossifications may be developing. A191 C3 1
191. A 43 year-old patient with a post myocardial infarction is on
A186 C6 1 digitalis to improve cardiac contractility. He is a new
186. A patient with a T4 spinal cord injury is being measured for participant in your Phase 2 outpatient cardiac rehabilitation
a wheelchair. In determining the correct seat height you program. He is being continuously monitored by ECG via
can use as a measure: radio telemetry. On his ECG, the medication induced
A. clearance between the floor and the foot plate of at changes that could be expected are:
least 2 inches. A. decreased heart rate with prolonged QRS and QT
B. clearance between the floor and the foot plate of at intervals.
least 4 inches. B. depressed ST segment with a flat T wave and
C. the distance from the bottom of the shoe to just under shortened QT interval.
the thigh at the popliteal fossa. C. elevated ST segment with T wave inversion.
D. the patient’s leg length measurement plus four D. widened QRS complex with a flattened P wave.
inches.
A192 C3 2
A187 C8 2 192. Your patient has lymphatic disease of the right arm
187. A 6 year-old boy has a diagnosis of Duchenne muscular secondary to radical mastectomy and radiation. The
dystrophy and is still ambulatory. The MOST appropriate resulting edema can BEST be managed in physical therapy
activity to include in his plan of care would be: by:
A. circuit training program. A. AROM and extremity positioning.
B. progressive resistance strength training. B. intermittent pneumatic compression, extremity
C. recreational activities such as swimming or biking. elevation, and massage.
D. wheelchair sports. C. isometric resistive exercises and extremity positioning
in elevation.
A188 C7 3 D. PROM and extremity elevation.

’05 Sullivan A
Practice Questions 19

A193 C7 2 important aspect of physical therapy intervention


193. You have been treating a patient for chronic subluxation of emphasizes:
the patella in the outpatient clinic. He is now scheduled for A. lumbar extension exercises.
a lateral release and is worried about any complications of B. modalities to decrease pain.
the surgical procedure. He asks you to describe any C. postural reeducation to prevent positions that
potential complications. Your BEST response is to: increase symptoms.
A. do a search on the internet and get back to the D. strengthening exercises for the abdominals and back
patient with the desired information. muscles.
B. explain how patients you have treated responded to
the surgery. A199 C6 1
C. refer the patient to a physical therapy colleague who 199. A patient with spastic left hemiplegia experiences severe
specializes in knee problems. genu recurvatum during stance phase. If the patient is
D. suggest that the patient speak with his surgeon. using an ankle-foot orthosis, the cause of the problem
might be attributed to:
A194 C1 3 A. the anterior stop setting the foot in too much
194. A 46 year-old female was referred for physical therapy dorsiflexion.
following a right breast lumpectomy with axillary lymph B. the anterior stop setting the foot in too much plantar
node dissection. Scapular control is poor when upper flexion.
extremity flexion or abduction is attempted. PT intervention C. the posterior stop setting the foot in too much
should focus on: dorsiflexion.
A. active assistive pulley exercises to assist rotator cuff D. the posterior stop setting the foot in too much
muscles as a result of damage to the suprascapular plantarflexion.
nerve.
B. gravity assisted right upper extremity exercises to A200 C3 3
promote scapular control following damage to the 200. A patient has arterial peripheral vascular disease with
long thoracic nerve. symptoms of intermittent claudication in both lower
C. rhomboid strengthening as a result of disuse of the extremities. Which of the following is NOT an appropriate
scapular stabilizers. guideline for exercise training?
D. strengthening of the right deltoids to help stabilize the A. daily walking through pain in order to attain
shoulder to compensate for damage to the dorsal predetermined distances.
scapular nerve. B. daily walking, 2 to 3 times a day.
C. interval training protocol with frequent rests.
A195 C3 3 D. stopping exercise and resting once the pain threshold
195. A post surgical patient is receiving a regimen of postural is reached
drainage three times a day. You could reduce the
frequency of treatment if the:
A. amount of productive secretions decreases.
B. consistency of the sputum changes.
C. patient becomes febrile.
D. patient experiences decreased postoperative pain.

A196 C2 1
196. The distinguishing feature of Mèniére’s disease which sets
it apart from other peripheral conditions affecting the
vestibular system is:
A. chronic, episodic bouts of dysfunction.
B. postural instability.
C. significant dysfunction with severe attacks of vertigo
and nausea.
D. time-limited, usually lasting only 24-48 hours.
A197 C7 3
197. A physical therapist arriving at work one hour before she
was due to start work, began moving treatment tables and
rearranging the physical therapy clinic. This operation
could have been done during regular hours. The therapist
sustained a low back injury as a result of moving the
equipment. Payment for the therapist’s care relating to this
incident would be covered primarily by:
A. employee’s health insurance.
B. the hospital’s insurance company.
C. the therapist’s own resources.
D. Workers’ Compensation.

A198 C1 2
198. Patients diagnosed with Paget’s disease typically have
similar symptomatology to spinal stenosis. The most

‘05 Sullivan A
20 Practice Questions

1. glenohumeral mobilization and strengthening of scapular intraoral appliance occurs only when the acute
stabilizers to regain normal scapulohumeral movement. inflammation is not resolved or bruxism continues. Joint
Compensation of glenohumeral restrictions if often mobilization should not be attempted with an acute
exhibited as excessive scapular movement. Therefore, inflammation.
mobilization of the glenohumeral joint and strengthening of
scapular stabilizers is needed to regain normal 10. posterior right C7 articular pillar. The most effective hand
scapulohumeral motion. placement for mobilization into great left rotation is at the
posterior aspect of the right C7 articular pillar because it
2. unilateral hearing loss. Presbycusis is a sensorineural rotates the C7 vertebra to the left.
hearing loss associated with aging. It is characterized by
bilateral hearing loss with deficits noted in auditory 11. transfer objects from one hand to another. Transferring
discrimination and comprehension. objects from one hand to another is a task developmentally
appropriate for an 8-9 month-old. Using a fine pincer grasp
3. increase the warm-up and cool-down periods to equal the and building a tower of 4 blocks are skills which develop
total aerobic interval in time. Clinical decisions should focus later. Holding a cup by the handle while drinking usually
on reducing the environmental costs of exercising (change occurs by 12 months of age.
the time of the day of the exercise class to reduce the heat
stress) or reducing the overall metabolic costs of the 12. document your observations and refer him back to his
activity (decrease the pace of the exercise, add more rest physician for evaluation of possible dopamine toxicity.
periods). Altering the warm-up and cool-down periods does Dyskinesias (involuntary movements) are a common side
not lower the overall cost of the aerobic exercise period. effect of dopamine toxicity as are gastrointestinal
4. anterior tibia, tibial crest, and fibular head. In the PTB disturbances (nausea, vomiting) and mental disturbances
socket, reliefs are provided for pressure sensitive areas: (restlessness, general overactivity, anxiety or depression).
the anterior tibia and tibial crest, fibular head, and peroneal While the symptoms described in the other choices may
nerve. All the other choices are considered pressure also occur with Parkinson’s disease, they do not
tolerant areas. adequately explain the presence of adventitious or
involuntary movements.
5. take time now to allow the patient to express her fears and
frustrations. It is important to be supportive of a patient who 13. explain how patients typically respond to the surgery and
is experiencing losses and resentment. Allow the patient to outline the progression of exercises. Assess the needs of
fully verbalize her feelings and frustrations. the patient and provide appropriate information based on
the expected rehabilitation process. Do not “pass the buck”
6. 1 MHz continuous at 1.0 W/cm2. 1 MHz of continuous unless the information is outside of the scope of your
ultrasound provides deep heating to a depth of 3-5 cm. At expertise.
this frequency, attenuation (absorption) is les in superficial
tissues. His allows more energy to be absorbed, thus more 14. patient reaches age-predicted maximal heart rate. A
heat production in deeper tissue layers. Continuous US is maximum exercise tolerance test is a sign or symptom
applied to achieve thermal effects (i.e., chronic pain) and limited test. Achieving age adjusted predicted maximum
pulsed US is used when nonthermal effects are desired heart rate is not a sign or symptom and therefore does not
(i.e., acute soft tissue injuries). stop the test.

7. some recovery of function since damage is to peripheral 15. swing-to. A swing-to gait pattern is indicated to individuals
nerve roots. A spinal cord lesion below L1 is a cauda with limited use of both lower extremities and trunk
equina lesion (injury to the peripheral roots and nerves). instability. It is slower and more stable than a swing-
Since some regeneration is possible, some recovery in through gait pattern (a gait pattern this patient can be
function can be expected. A spastic or reflex bladder is progressed to after his initial training). This patient is
associated with upper motor neuron injury. Other choices unable to perform a reciprocal gait.
describe the deficits associated with anterior cord
syndrome or central cord syndrome. 16. request her primary physician to refer her for psychological
evaluation. The patient is experiencing grief over her loss.
8. be designated as a more advanced task and more Significant persistent symptoms are an indication for a
appropriately delegated to another physical therapist. referral to a qualified professional (psychologist) to help her
Physical therapy students should not perform advanced deal with her loss.
tasks for the first time without any instruction or direct
supervision. This might be unsafe for the patient. The task 17. friction. Friction massage is effective when applied across
of orthotic checkout is most appropriately performed by (perpendicular) muscle fibers to stretch scars and loosen
another physical therapist. adhesions in the tissue due to the inflammatory process.
Stroking is a relaxation technique and usually initiates and
9. instruction to eat a soft food diet and phonophoresis. ends massage treatment. Some passive muscle stretching
Phonophoresis and education regarding consumption of is performed with deep stroking. Kneading aids in
only soft food with help resolve the acute inflammatory loosening adhesions and increasing venous return.
process in the temporomandibular joint. Application of an

’05 Sullivan A
Practice Questions 21

Tapotement is used when nerve stimulation or lung 27. tight iliopsoas muscle. Tight iliopsoas muscles are
decongestion is the desired treatment effect. common after THR surgery because the patient typically
spends more time in a sitting position. The iliopsoas
18. vigorous range of motion of the lower extremities. attaches to the transverse processes of the lumbar spine
Vigorous range of motion is contraindicated due to and the lesser trochanter of the femur. Therefore, when the
osteoporosis and risk of fracture. All of the other choices iliopsoas is tight, the lumbar spine will extend if the femur is
are appropriate to promote functional independence. fixed. In supine, the hamstrings and piriformis muscles are
on slack and would not cause alteration in spinal position.
19. SI joint. Pain at end range of flexion, abduction, and
external/lateral rotation (FABERE Test) is diagnostic for SI 28. environmental changes, a bedside commode, and referral
joint dysfunction because it both gaps and compresses the for home health services. Clinical decision making in this
joint. Pain at the midrange into hip flexion, abduction and case should focus on the patient’s ability to manage in the
external/lateral rotation suggests hip joint pathology. home. Environmental modifications (the addition of a
commode) and assistance of a home care aide should
20. chi square test. Chi square is nonparametric statistical test allow her to safely return home. The referral for home
used to compare data in the form of frequency counts (in physical therapy should focus on improving her endurance
this example frequencies of carpal tunnel syndrome) to regain independence in the home. Treatment at home is
occurring in two different groups of workers. the most cost effective in this case.

21. median. The mean is a measure of central tendency that is 29. make the patient aware of his deficit and teach him to turn
calculated by adding up all the scores and dividing the total his head to the affected left side. A patient with
by the number of scores. The median is a middle score homonymous hemianopsia needs to be made aware of his
while the mode is the most frequently occurring score. The deficit and instructed to turn his head to the affected left
most accurate measure of performance is skewed side (a compensatory training strategy). Initial strategies
distribution with extreme scores in the median. include placing items or doorway on his right (unaffected
side) so he can successfully interact with his environment.
22. denervation atrophy has occurred. Spontaneous fibrillation Later, as he demonstrates the ability to compensate, items
potentials present on EMG are evidence of denervation can be moved to midline and finally to his affected left side.
atrophy. Polyphasic motor units of low amplitude and short
duration are evidence of reinnervation. 30. reduce the exercise intensity and provide relaxation
strategies. Common problems in multiple sclerosis include
23. one-arm drive chair. A one-arm drive wheelchair has both fatigue and heat tolerance. The agitation and irritation are
drive mechanisms located on one wheel. The patient can most likely a result of frustration and fatigue. Exercise
propel the wheelchair by using one hand(in this case his intensity should be reduced and relaxation strategies
sound left hand). It is contraindicated in patients with utilized. A cool environment is important to reduce fatigue
cognitive or perceptual deficits. A hemiplegic chair (low but would not address the problems of agitation and
seat height) allows use of both the sound hand and leg to irritation. A warm pool is contraindicated.
propel the chair. The electric wheelchair with joystick might
also work but is significantly more expensive, less 31. Medicare. Custodial care of a medically stable individual
transportable, and would require increased maintenance. (over 65) who needs assistance with basic activities of
An elevating legrest is needed to complete the wheelchair daily living such as eating, dressing, toileting and bathing in
prescription. the home is covered by Medicare only when skilled care
(the services of a professional nurse or therapist under a
24. functional residual capacity. Muscular dystrophy will alter physician-authorized plan of home care) is required.
the muscles’ ability to pull in air and blow out air; therefore,
vital capacity, total lung capacity, and forced expiratory 32. instruct in proper dressing changes and wound care.
volume in one second will be decreased. Since muscular Proper dressing changes and wound care to prevent
dystrophy does not change the lung parenchyma, REEP secondary wound infection are the hallmarks of
will occur at same point of equilibrium between lung recoil conservative management of venous ulcers. Pliable,
and thoracic outward pull. Therefore, functional residual nonstretchable dressing wraps (e.g., Unna boot) or
volume will not change. custom-fitted support stockings can be used to assist in
venous circulation while elastic wraps do little to assist
25. swing phase. Foot drop is a swing phase deficit. circulation. Prolonged hydrotherapy is contraindicated for
Stimulation of the dorsiflexor muscles during the swing venous ulcers.
phase places the foot in a more neutral position and
prevents the toes from contacting the ground and 33. spasticity or contracture of the plantarflexors. Forward
interfering with the gait pattern. advancement of the tibia from midstance to heel-off is
controlled by eccentric contraction of the plantarflexors;
26. refuse to treat that patient. Blood and Body Fluid from heel-off to toe-off the plantarflexors contract
Precautionary Guidelines from the Centers of Disease concentrically. Either spasticity or contracture of the
Control (CDC) state that a healthcare worker with plantarflexors would limit this forward progression. Patients
exudative lesions or weeping dermatitis should refrain from compensate by going right into swing, typically with
all direct patient care and from handling patient-care circumducted gait, or with increased hip and knee flexion
equipment until the condition resolves. since there is no push off.

‘05 Sullivan A
22 Practice Questions

34. thumb abduction. Thumb abductors are innervated by the 44. training all staff to do simple repairs on all electrical
median nerve, primarily by the C6 nerve root. The anterior equipment if a breakdown should occur. Electrical
divisions contribute to nerves that primarily serve flexors, equipment is repaired by the manufacturer or local vendor,
and in this case, the thumb. or in some cases, the maintenance department, not by
physical therapy staff.
35. be calm and supportive, using only one or two level
commands. An agitated patient with dementia does not 45. cellulitis. Cellulitis is an inflammation of the cellular or
process information easily. A calm and supportive approach connective tissue in or close to the skin. It is characterized
with low level commands (one or two actions) provides the by skin that is hot red, and edematous. Fever is a common
best approach to take with this patient. finding. Dermatitis produces red, weeping, crusted skin
lesions, but is not commonly accompanied by fever.
36. postural hypotension. Fall risk is significantly increased Location on shins makes herpes an unlikely choice and
with certain medications. Tricyclic antidepressants (e.g., there are no skin eruptions, or vesicles. Scleroderma is a
Elavil) can be effective in relieving depression but may collagen disease producing tight, drawn skin.
cause postural hypotension, fainting or confusion, thus
increasing fall risk. The elderly are particularly susceptible 46. ultrasound. Ultrasound energy may affect the epiphyseal
to adverse drug effects due to multitude of factors. areas causing bone growth disturbances. The epiphyseal
plates close at the end of puberty.
37. weakness and palpitations. Digitalis (Digoxin) is frequently
used to treat congestive heart failure (it slows heart rate 47. 6-8 weeks of training. Hypertrophy is the increase in muscle
and increases force of myocardial contraction). Adverse size as a result of resistance training and can be observed
side effects of digitalis can include muscle weakness and following at least 6-8 weeks of training. Individual muscle
supraventricular or ventricular arrhythmias including fibers are enlarged, contain more actin and myosin, and
ventricular fibrillation without premonitory signs. have more, larger myofibrils.

38. a treadmill purchase. Any purchases of $300 or more 48. external/lateral rotation and extension. The long head of
would be considered a capital expense and listed as an the biceps is best exposed in shoulder lateral rotation and
expense in the capital budget, not an operating budget. extension due to its attachment at the supraglenoid
tubercle of the scapula which is at the medial aspect of the
39. wear gloves if there is direct contact with blood or body shoulder joint. Medial rotation and abduction places the
fluids. Standard Precautions specify health care workers long head of the biceps deep to the anterior deltoid and
wear gloves when they come into direct contact with blood or pectoralis major muscles. The anterior surface of the
body fluids. Health care workers should wear moisture- shoulder, including the long head of the biceps, loses
resistant gowns and masks for protection from the exposure with horizontal adduction.
splashing of blood, other body fluids or respiratory
droplets. 49. step up onto a low step while in the parallel bars. Active
practice of stepping using a low step represents the best
40. continuous monophasic current with the medication under choice to ensure motor learning. Passively bringing the loot
the anode. Continuous unidirectional current flow is more up does not promote active learning. Choices A and C are
effective in repelling ions into skin than pulsed or appropriate lead-up skills to stair climbing.
bidirectional current. Procaine is a positive medicinal ion
and will be repelled from the anode (positive pole). 50. tandem walking and single limb stance. A patient with
balance instability would benefit from all of the
41. long term corticosteroid therapy. Very often patients with interventions except tandem walking and single limb
chronic pulmonary disease have been managed using stance. These activities are too difficult (high level) to be
corticosteroid therapy. Long term steroid use affects included in an initial training program. A significant
ligamentous integrity, which often produces joint percentage of healthy elderly are unable to stand on one
hypermobility. leg.

42. Medicaid Long term care for institutionalized elderly is 51. 252 inches (21 feet). The architectural standard for rise of a
funded by Medicaid, a state-federal partnership of funding. step is 7 inches (steps may vary from 7-9 inches). The
Patients must first spend down their assets to qualify for low recommended ratio of slope to rise is 1:12 (an 8% grade).
income stains before they are eligible for Medicaid. Individual For every inch of vertical rise, 12 inches of ramp will be
slates determine eligibility requirements. required. A straight ramp will have to be 252 inches or
21 feet long.
43. infected pleura. The case is supportive of a pulmonary process
as evidenced by radiography and history. Since the radiographic 52. increased tidal volume. Asymmetrical breathing, deviated
findings and the pain are in the same vicinity, the likelihood is this trachea towards the side to the pneumonectomy, and
pain is pleuritic in origin. Angina is not the most likely cause since decreased breath sounds would be likely changes. An
the cardiac system is not involved. There is no history of trauma increased tidal volume due to incisional pain and sedation
to the chest and no trauma was found radiographically, making would not be likely.
it unlikely as the source of pain. An inflamed tracheobronchial
tree would not usually reflect pain in the posterior base of the 53. shortness of breath at rest and with limited activity, and
left thorax. sudden weight gain. Elderly patients with moderate to
severe congestive heart failure demonstrate diminished
cardiac reserve and intolerance to strenuous physical

’05 Sullivan A
Practice Questions 23

activity. Symptoms of exertional intolerance include decision and is based on additional factors such as age,
shortness of breath and sudden weight gain, the result of balance, strength, cognition and so forth.
circulatory backflow and peripheral edema.
62. 5 times/week, BID 10 minutes/session. Patients with
54. position in prone lying and sitting with full knee extension. vascular insufficiency and claudication pain should be
The typical contractures with a transtibial amputation are encouraged to walk daily, 2-3 x/day. Duration should be short.
knee flexion and hip flexion (typically from too much sitting). The patient should walk to the point of maximum tolerable
When in bed, hip and knee extension should be emphasized pain, and allowed to rest.
(e.g. prone-lying). When sitting in the wheelchair, knee
extension should be emphasized (e.g. using a posterior 63. oblique. The spinal defect is spondylolisthesis and the
splint or knee board). radiographic view that demonstrates the scotty dog neck
fracture is the oblique view. A lateral radiographic view will
55. postpone therapy and coordinate with the nurse regarding show the degree of anterior or posterior slippage of one
insulin management and exercise. Normal lasting plasma vertebra on another allowing the radiologist to grade the
glucose is less than I 15 mg/dL while a fasting plasma spondylolisthesis.
glucose level greater than 126 mg/dL on more than one
occasion is indicative of diabetes. This patient is 64. allow the patient to express her anger while refocusing her
hyperglycemic with high glucose levels (equal to or on effective coping strategies. Anger is a recognized stage
greater than 250 mg/dL). Clinical signs that may accompany in the psychological adaptation to death and dying (Kubler-
this condition include ketoacidosis (acetone breath) with Ross). Patients should be allowed to express anger,
dehydration, weak and rapid pulse, nausea/vomiting, deep and frustration, and resentment. A helpful strategy is to redirect the
rapid respirations (Kussmaul’s respirations), weakness, dimin- patient to achieve effective coping strategies (anger
ished reflexes, and paresthesias. The patient may lie management techniques). While honest, accurate
lethargic and confused and may progress lo diabolic coma information is important it is not the most useful strategy for
and death if not treated promptly with insulin. Physical this patient at this time.
therapy intervention is contraindicated; exercise can lead to
further impaired glucose uptake. 65. a facet joint. Facet joint dysfunction is exacerbated with
sustained positions and eases with movement.
56. splinting the shoulder in abduction and internal rotation.
Splinting the shoulder in abduction leads to formation of 66. document the findings and immediately inform the patient’s
abduction contractures and later hypermobility of the physician about the situation. It is most appropriate to
shoulder. discuss a patient problem with the person who is ultimately
responsible for the patient. The physician as team leader
57. extension, left side-bending, and right rotation. The right can affect a change in behavior or seek additional help
sternocleidomastoid produces left lateral rotation and flexion of for the patient.
the cervical spine. The right sternocleidomastoid is in
lengthened position with the head turned to the right and the 67. forward step-ups in standing using graduated height steps.
cervical spine extended. Decreased foot clearance during swing may result from
weak hip and knee flexors or may be the result of a drop
58. he should be expected to value patient confidentiality. foot (weak dorsiflexors or spastic plantarflexors). Step-ups
Valuing and upholding patient confidentiality is an expected represents the best choice to functionally strengthen the
behavior for all physical therapists and physical therapy hip and knee flexors. Bridging promotes knee flexion with
students specified by the American Physical Therapy hip extension. The sitting activities promote hip abduction
Association Code of Ethics. Since this a final affiliation, the (A) and knee extension (B).
student should adhere fully to the APTA's Code of Ethics.
68. a reciprocating gait orthosis and walker. A patient with a
59. B is less than A. In an A-B-A-B single subject design, the As SCI at the level of L1 or above has lost all functioning
represent multiple baseline-measurements while the Bs lower extremity muscles needed for gait. A
represent multiple post-treatment measurements. If the reciprocating gait orthosis is the best choice for this
hypothesis is accepted, the pain rating scores will he lower patient. Because of the high energy expenditure, patients
following the treatment when compared to the baseline with injuries at this level typically depend upon the
measurements. wheelchair as their primary means of mobility. However,
the physiological and psychological benefits make it is
60. improve rate control at faster movement speeds. Patients appropriate to train this patient in ambulation.
during the later stages of recovery from stroke frequently
exhibit problems with rate control. They are able to move 69. appropriate if intensities are kept below 40% maximal
at slow speeds but as speed of movement increases, control voluntary contraction. Resistance training is typically
decreases. An isokinetic device can be an effective training initiated after patients have completed 4 to 6 weeks of
modality to remediate this problem. supervised cardiorespiratory endurance exercise. Lower
intensities are prescribed. Careful monitoring of BP is
61. use a shrinker. A shrinker is a suitable alternative to necessary as BP will be higher and HR lower than for
elastic wraps. It is important to select the right size aerobic exercise.
shrinker to limit edema and accelerate healing. An Unna's
paste dressing is applied at the time of initial surgery. Use of 70. change the angle of pull. Changing the angle of pull will
a temporary prosthesis should be a prosthetic team reduce the traction pressure on the mandible and at the

‘05 Sullivan A
24 Practice Questions

TMJ. The greatest traction force should be felt on the and hold-relax techniques can be used to improve flexibility but are
occiput. also not likely to be effective in this case because of the short
duration.
71. assisted breathing and coughing and pelvic floor exercises.
It is important to teach the patient to provide incision support 81. stretching and limiting extended spinal positions as the
for breathing and coughing. Pelvic floor exercises are also result of spinal stenosis. Spinal stenosis presents with
important since hours of labor and pushing arc typically bilateral dysesthesias and pain in extended positions
present before surgery. Assisted ambulation and ankle and/or during walking for distances greater than 100 feet.
exercises would routinely be carried by nursing personnel Physical therapy intervention should emphasize stretching
and would not necessarily require a P.T. referral. of tight structures and dynamic control of the trunk to limit
long-term extended spinal positions.
72. spondylolisthesis with possible anterior slippage of the
vertebral body. Spondylolisthesis is a bilateral fracture of the 82. have the teacher give a smile sticker when the child sits
pars interarticularis with anterior slippage. Spondylolysis is a with head retracted for five minutes. Positive reinforcement
unilateral or bilateral fracture without anterior slippage. (use of the smile sticker) is an effective way to shape
behavior by operant conditioning. Negative behaviors are
73. adapt a desk and wheelchair to provide adequate sitting ignored (slumping in the chair) while positive behaviors are
balance. The goal of school physical therapy is to directly encouraged (hold head in a retracted position for 5
facilitate the educational process, for example, interacting in minutes.
class viewing the blackboard, etc.
83. ask your colleagues about their current level of knowledge
74. a rolling walker to compensate for impaired balance. The using a brief questionnaire. In order to better share the
patient with Parkinson's disease typically presents with information you have learned, you need to assess what
postural deficits of forward head and trunk with hip and knee information and skills your colleagues currently have. A
flexion contractures. Gait is narrow-based and shuffling. A brief questionnaire can be an effective means to achieve
festinating gait may result from persistent forward posturing this. Choices A, B, C, demonstrate planning of the learning
of the body near the forward limits of stability. A rolling experience without benefit of a needs assessments.
walker is contraindicated because it would increase
forward postural deformities and festinating gait. 84. passive manipulation to the shoulder. All of the treatments
may lie used for RSD (Complex Regional Pain
75. ideomotor apraxia. With ideomotor apraxia, a patient Syndrome, CRPS) except passive manipulation which
cannot perform a task upon command but can do the task may aggravate sympathetically maintained pain.
when on their own. With ideational apraxia, a patient
cannot perform the task at all. 85. lower trapezius, latissimus dorsi, and triceps. The upper
quadrant muscles that arc most important to strengthen
76. should be paid for by a third party payer. Determination for crutch gaits include the lower trapezius, latissimus
about whether care is paid for by a third party payer is dorsi, and triceps. Shoulder depression and elbow
determined by the payer based on contract terms and extension strength is crucial.
condition. Ideally those terms include same or similar clinical
factors and rationale that are part of appropriate peer review 86. 92%. A 92% SaO2 is low but acceptable.
but they need not be. Unacceptable oxygen saturation rates during exercise
are < 88%. Normal SaO2 is >95% - 98%.
77. increased total lung capacity. An obstructive pattern on
pulmonary function tests includes increased total lung capacity 87. ice massage B.I.D. until the pain subsides. Pain and
caused by destruction of alveolar walls. This same destruction inflammation can be effectively treated with a cold modality.
causes an increased residual volume with a resulting increased Ice massage can rapidly reduce pain and tissue
functional residual capacity and decreased vital capacity. temperature in a small area without affecting the bony
structure. Treatment frequency should increase if pain is
78. ischial tuberosity, gluteals, and lateral sides of residual severe. Fluidotherapy three times a week is not
limb. A quadrilateral socket in a transfemoral amputation is appropriate.
designed to selectively load tissues that are pressure tolerant:
the ischial tuberosity, gluteals, and lateral sides of the residual 88. side-bending. The uncinate processes (joints of Luschka)
limb. are located-at the inferior lateral aspect of the lower
cervical vertebrae. Side-bending is lost with
79. immediately contact her primary physician. This patient is degenerative changes at the joint the uncinate process
presenting with signs of clinical depression. Her primary physician makes with the vertebra below. Other motion is
should be contacted immediately (especially if suicide is restricted but to a lesser degree.
mentioned).
89. active assistive pulley exercises. The patient is most likely
80. mechanical stretching using traction and 5 lb. weights, 2 suffering from a rotator cuff tear. Acute physical therapy
hours, twice daily. Prolonged mechanical stretching involves intervention should focus on reduction of pain and
a low-intensity force (generally 5 to 15 Ibs.) applied over a inflammation. During the early subacute phase, active
prolonged period (30 minutes to several hours). It is generally assistive pulley exercises would be indicated lo promote
the most effective way to manage long-standing flexion healing of the supraspinatus muscle and maintain active
contractures. Manual passive stretching is a short duration stretch range of motion of the glenohumeral joint.
that is not likely to be effective in this case. Lower extremity splints

’05 Sullivan A
Practice Questions 25

90. interrater reliability. Interrater reliability is the degree to and the therapist should don a tight fitting mask prior to
which two or more independent raters can obtain the same entering the room. The patient only needs to wear a mask
rating for a given variable. In this case, two therapists if he needs to leave his room (for a medical test, etc.). See
obtained different HIM scores for the same group of Transmission-Based Precautions, Table 5-3.
patients, indicating a problem in interrater reliability.
100. patient education regarding avoidance of squatting and
91. sit the patient up, check/empty catheter, and then call for jumping activities as well as initiation of iontophoresis
emergency medical assistance. The patient is exhibiting using dexamethasone. The dysfunction observed on the
autonomic dysreflexia (an emergency situation). You x-ray is Osgood-Schlatter's Disease. The radiograph
should first sit the patient up and check for irritating or depicts epiphysitis of the tibia at the attachment of the
precipitating stimuli (e.g., a blocked catheter). Then call for patellar tendon seen in adolescents. It occurs as the result
emergency medical assistance. of activities that require continued explosive contractions
of the quadriceps muscle complex during pubescent
92. metatarsal bar. Correction for a pes planus deformity growth spurts. Patient education should locus on control-
(support of the longitudinal arch) can be provided by a ling knee loading activities such as squatting and
UCBL insert, scaphoid pad or Thomas heel. A metatarsal jumping. Explosive contractions of the quadriceps
bar is indicated to take pressure off the metatarsal heads complex should be avoided. Ambulation and AROM
and improve push off. activities maintain mobility while the structure heals,
Phonophoresis would be contraindicated because it may
93. custom made pressure garments. Following burns, edema be painful to move the sound head over the affected
and hypertrophic scarring can be effectively controlled with area. Additionally, ultrasound should not be used over
elastic or pressure garments. Surgery is an option of last open epiphyses. Open chain knee extension exercise
resort. may aggravate symptoms due to the increased load at
the attachment of the patellar tendon to the tibial
94. dyspnea, anxiety, or disorientation. A patient with tuberosity.
respiratory acidosis may present with symptoms of
dyspnea, anxiety, restlessness, or headache. Significant 101. provide a two-wheel handcart for use in moving the boxes.
acidosis may lead to disorientation, stupor, or coma. Implementation of an engineering control technique
Answers A, B, C are signs and symptoms of respiratory can be accomplished by designing or modifying the
alkalosis. workstation, work methods, and tools to
eliminate/reduce exposure to excessive exertion,
95. placing the wrists in resting splints. Modalities which use awkward postures and repetitive motions.
steroids are contraindicated for pregnant women. The most
effective intervention would be to place the wrists in a neutral 102. subtalar pronation and medial rotation of the tibia. In order
position in splints. The carpal tunnel is, therefore, not to maintain the center of gravity over the base of support,
compromised by poor hand positioning while at work. the subtalar joint must pronate and the entire lower quarter
must medially rotate.
96. wheelchair with a back wedge and head supports. A
wheelchair wedge and head supports holding the trunk and 103. empty end-feel. An empty end-feel (no real end-feel) may
head in slight flexion will help decrease the extensor tone be indicative of severe pain and muscle guarding associated
and is the most appropriate positioning in the educational with pathological conditions. Springy and firm end-feels may
setting. An abduction pommel controls scissoring of the be expected after elbow surgery. Soft end-feel is an indication
legs but does not control head and upper trunk necessary of range limited because of tissue compression (e.g. in
for functioning in the school environment. knee flexion there is contact between the posterior leg and
the posterior thigh).
97. shoulder extensors, external rotators, and anterior deltoid
to position and lock the elbow. The patient with complete 104. behind and to the left side, one hand on the gait belt. The
C^, quadriplegia will lack triceps. He can be taught to lock correct guarding technique to protect a patient from failing
the elbow for push-up transfers by using shoulder external is to stand slightly behind and to one side (the involved or
rotators, extensors to position the arm; the anterior deltoid left side).
locks the elbow by reverse actions (all of these muscles are
functional). 105. unaffected. The chronaxie is the duration of the current at
twice the rheobase (minimal intensity at a given long
98. demonstration, practice, and follow-up discussion. A duration). The chronaxie of an innervated nerve is less than
variety of teaching methods including demonstration, then 1 ms. When a motor nerve is severed it undergoes a
practice, and discussion has the best chance of reinforcing process called Wallerian degeneration in which the axons
the learning in a diverse group. One teaching method is not distal to the lesion degenerate. The chronaxie of the
likely to be as successful in meeting the needs of all group denervated nerve is greater than I ms. During the first 7 to
members. 10 days much of the nerve constituents are st i l l present,
thus no change in the chronaxie is observed.
99. have the patient wear a tight fitting mask while being
treated in his room. The therapist should wash his/her 106. swimming using a crawl stroke. Positioning in spinal
hands upon entering and leaving every patient’s room. extension increases symptoms in patients with spinal
When the patient is suspected of having tuberculosis, the stenosis. Activities such as swimming using a crawl stroke
patient should be in a private, negative pressurized room. place the spine in a constant extended position. All other
The room is considered a potentially infective environment

‘05 Sullivan A
26 Practice Questions

activities described do not require the patient to maintain thickening, coarsening, and brownish pigmentation of the
an extended spinal position. skin around the ankles. The skin is usually thin, shiny, and
cyanotic. DVT may be asymptomatic initially. When
107. a shoe lift on the sound side. This patient is vaulting (rising symptoms occur, patients typically report a dull ache,
up on the sound limb to advance the orthotic limb forward). tightness, or pain in the calf.
This gait deficit increases the energy demands of gait and is
most likely contributing to his fatigue. The myalgia and 116. large electrodes, widely spaced; 10:30. Large electrodes
fatigue arc also direct impairments of post-polio syndrome. are used on large muscles in order to disperse the current
An appropriate intervention plan would be to reduce the (minimize current density under the electrode) enabling a
energy demands of gait. The use of a shoe lift on the sound more comfortable delivery of current. Widely spaced
side would help accomplish this. A wheeled mobility device electrodes permit the current to travel deeper into the
(motorized cart) would also assist in reducing energy muscle to stimulate a greater number of deeper muscle
demands of functional mobility. The wheelchair choices fibers. A duty cycle of 1:3 will allow a sufficient rest period
presented are poor ones. between contractions to minimize the fatiguing effect of
repetitive muscle contractions.
108. stimulate the ipsilateral triceps. Stimulation of the
ipsilateral triceps will reduce the muscle tone of the 117. call the physician immediately and report your findings. A
biceps through reciprocal inhibition. mole with irregular edges and multiple colors (black or
blue) is characteristic of a melanoma. It is associated with
109. ankle-foot orthosis. Individuals with an L5 lesion need an sun-damaged skin but can appear anywhere on the body.
orthosis to correct foot alignment and for push-off. There is Early recognition before tumor invasion and metastatic
partial innervation of the hamstrings and gluteus minimus spread is critical in improving survival rates. The physician
and medius, posterior tibialis and peroneus tertius muscles. should be informed immediately. Medical management
The other orthoses would he indicated for higher level consists of biopsy and surgical removal.
lesions.
118. decreasing consciousness with slowing of pulse and
110. closed-chain partial weight-bearing lower extremity Cheyne-Stokes respirations. Signs of increased intracranial
exercises for slipped capital femoral epiphysis. Slipped pressure secondary to cerebral edema and brain herniation
capital femoral epiphysis is characterized by a glutens include decreasing consciousness with slowing of pulse and
medius gait. Closed-chain exercises with weight-bearing to Cheyne-Stokes respirations. Cranial nerve dysfunction is
tolerance will help regain or maintain functional muscular typically noted in C.N. II (papilledema) and C.N.III (dilation
strength and normal motion. of pupils). Choices A and C are signs of meningeal
irritation and CNS infection.
111. null hypothesis. The null hypothesis is a statistical
hypothesis that states that there is no relationship (or 119. nerve dysfunction will be rapidly reversed, generally in 2-3
difference) between variables. Any relationship found will weeks. Neurapraxia is a mild peripheral nerve injury
be a chance relationship, not a true one. (conduction block ischemia) that causes transient loss of
function. Nerve dysfunction is rapidly reversed, generally
112. that the therapist was functioning outside the common within 2 - 3 weeks. An example is a compression injury
protocols of the hospital, and therefore did not support the to the radial nerve from falling asleep with the arm over
actions of the physical therapist. The physical therapist the back of a chair (Saturday night palsy).
was acting outside of her area of responsibility and did
something that caused the patient harm. 120. occasional feedback given when consistent errors appear.
In learning a psychomotor skill, the patient needs to be able
113. patch one eye. Double vision (diplopia) can be managed by to actively process information and self-correct responses.
patching of one eye. Patients are typically on an eye Occasional feedback provides the best means of allowing for
patching schedule which alternates the eye that is patched. introspection and is appropriate for later in practice
Loss of depth perception can be expected with eye patching (associated and autonomous phases of motor learning).
but is not as disabling as diplopia.
121. he may be presenting with early signs of myocardial
114. sidelying on the sound side with the affected upper infarction. An elderly patient with a cardiac history may
extremity supported on a pillow with the shoulder present with initial symptoms of mental confusion, the result
protracted and elbow extended. Most patients with stroke of oxygen deprivation to the brain. The SOB and
recover from the flaccid stage and develop spasticity. generalized weakness may also be due to generalized
Positioning for the patient with early stroke stresses (I) circulatory insufficiencies coexisting with the developing
protection against ligamentous strain and the development of a myocardial infarction.
painful subluxed shoulder and (2) positions counter to the
typical spastic posture of flexion and adduction with pronation. 122. modalities to reduce inflammation, active assistive range of
Sidelying with the affected upper extremity supported on a motion exercises using pulleys, and postural realignment.
pillow with the shoulder protracted and elbow extended Initial physical therapy intervention of tendinitis with possible
accomplishes both of these goals. The other positions do not. impingement should emphasize active assistive exercises,
modalities to reduce inflammation, and postural
115. chronic venous insufficiency. Venous ulcers are often realignment to reduce impingement of the rotator cuff at the
painless, or present with minimal pain when compared to acromion.
arterial ulcers which are painful (claudication and rest pain).
Chronic venous insufficiency is also characterized by

’05 Sullivan A
Practice Questions 27

123. 10 to 15 seconds. The recommended time duration for findings, and determine the physical therapy diagnosis.
endotracheal suctioning is 10 to 15 seconds. Any longer While many states have direct access laws which permit
risks serious any shorter and the risk is ineffective physical therapy intervention without referral, most
secretion removal. insurance companies including Medicare (affecting the
patient in this example) require a physician referral in order
124. occlusive dressings. Autolytic wound debridement allows for services to be reimbursed. Thus the therapist needs to
the body's natural enzymes to promote healing by trapping consult with the physician to get a referral before initiating any
them under a synthetic, occlusive dressing. The dressings intervention for this problem. This patient demonstrates
arc applied for short durations (less than 2 weeks ) and arc stress incontinence, a problem that could be successfully
contraindicated in infection. The other answer are treated with physical therapy (e.g., Kegel exercises and
appropriate wound management techniques; however, they other interventions).
are not autolytic.
132. anterior spinothalamic tract. Sensations interpreted as dull,
125. switch to modulation mode TENS. Because of the long- aching pain travel in the anterior (paleo) spinothalamic
term continuous use of TENS, the sensory receptors tract. Discriminative fast pain is carried in I he lateral (neo)
accommodated to the continuous current and no longer spinothalamic tract. Discriminative touch is carried in the
responded to the stimuli. Changing to modulation mode (i.e. proprioceptive pathways (fasciculus gracilis/cuneatus, medial
burst modulation), which periodically interrupts the current lemniscus).
flow docs riot allow accommodation to occur.
133. increase the patient’s maximal oxygen consumption by
126. gastrocnemius-soleus. The muscles of the foot and ankle discharge. All of the choices are important goals for Phase
move the long lever of the body forward and backward I, acute or inpatient cardiac rehabilitation except for altering
using ankle synergies. The gastrocnemius-soleus moves maximal oxygen consumption. Patients generally are
the body backward while the anterior tibialis moves the expected to reach a functional capacity of 3-5 METs by
body forward. Action of the hip extensors would result in a discharge from a Phase I program.
backward lean with the center of motion occurring at the
hip. 134. placement of the drive wheels 2 inches posterior to the
vertical back supports. Placement of the drive wheels 2
127. false negative. Sensitivity refers to the ability of a test to inches posterior to the vertical back supports is an
correctly identify individuals who truly have a disease or appropriate modification for a patient, with bilateral
condition (a true positive). In this example, the individual transfemoral amputations. This increases the length of the
was found not to have the Alzheimer's gene, when years base of support and provides increased posterior stability.
later he tested positive for the gene and the disease. The Lowering the seat height by 3 inches is an appropriate
original test produced a false negative result. modification for a patient with stroke who will use his sound
limbs for wheelchair propulsion. Increasing the seat depth
128. history of sudden onset of new cognitive problems and is not an appropriate modification.
patchy distribution of deficits. Multi-infarct dementia differs
from primary degenerative dementia, Alzheimer's type, in 135. P=0.05. A preselected probability level of 0.05 indicates
(1) onset: sudden rather than slowly progressive that the results (in this example differences in
(characteristic of small focal infarcts) and (2) the nature of measurements) would be the result of chance only 5 times
symptoms: areas of deficits coexist with areas of intact out of every 100 studies. This level of confidence helps us
cerebral function. Agitation and sundowning (late afternoon reject the null hypothesis (there is no difference in
wandering) are characteristics of Alzheimer's disease. goniometric measurement techniques) and is common in
most experimental studies (0.01 is the other, more stringent
129. vastus medialis muscle strengthening. Q angles greater level of significance commonly applied).
than 15 degrees could be indicative of abnormal lateral
patellar tracking. Vastus medialis muscle strengthening can 136. instruction in proper postural alignment. Without regaining
reduce the tendency for the patella to track laterally. Vastus normal postural alignment and scapular-humeral rhythm,
lateralis (VL) strengthening can promote greater lateral the patient will continue to impinge the supraspinatus
patellar tracking and further irritation of the patellofemoral and/or biceps tendon at the acromion and never regain
joint. VL strengthening may promote an outward pull or normal function of the shoulder. It is unlikely that all pain
dislocation of the patella. Hamstring strengthening does not would be controlled.
directly affect tracking of the patella.
137. pain and stiffness are worse in the early morning and
130. standing, picking the foot up behind and slowly lowering it. should decrease with moderate activity. Degenerative joint
Stage 4 recovery is characterized by some movement disease (osteoarthritis) is a noninflammatory progressive
combinations that do not follow that paths of either flexion or disorder affecting primarily the carpometacarpal, knee, and hip
extension synergies. Knee flexion in standing is an out-of- joints. Stiffness is common following inactivity and is
synergy movement. All other choices represent synergistic relieved with movement. Exercise can be helpful in reducing
movements: choices A and B are flexion synergy pain and immobility and is not contraindicated. Joint projection
movements while choice C focuses on knee extensor strategies are effective intervention for maintaining function.
movement within an extended position.
138. symmetrical distribution of weakness, ascending with
131. examine the patient, document and discuss your findings possible involvement of lower cranial nerves. Guillain-
with the doctor. The physical therapist should complete the Barre syndrome is an acute polyneuritis characterized by
examination of the patient, adequately document the rapid development of progressive muscle weakness. The

‘05 Sullivan A
28 Practice Questions

weakness is typically symmetrical and ascends the body 147. supine, head of bed flat. A patient with a C5 spinal cord
(starting first in the lower extremities, progressing to trunk, injury will not have the abdominal musculature necessary
upper extremities, and finally cranial nerves). Stocking and to return the diaphragm to a high domed position during
glove sensory loss could also be found but not with minor exhalation. Inspiration will be affected by the change in the
loss of motor function. diaphragm's resting position. In supine, gravity will take the
place of abdominals, holding the abdominal contents
139. monophasic current with the anode placed on the tendon. under the diaphragm, improving the zone of apposition,
The anode (positive) electrode would effectively repel the the height of the diaphragm dome and therefore, the
positively charged ion into the tendon. Because of its ability to ventilate. The other positions listed negate the
unidirectional flow, monophasic currents are best suited positive effects of gravity on the abdomen.
for iontophoresis.
148. schedule a conference with the doctor and family about her
140. have the patient do sitting push-ups at least every 10 condition and your discussions with the patient. The most
minutes. Excessive ischial pressure and redness from appropriate strategy is to hold a conference with the doctor
prolonged sitting requires an aggressive approach. Arm and family about her condition and your discussions with the
push-ups, at least every 10 minutes, is indicated if redness patient. Everyone interacting with this patient should be
is present. A high-density gel cushion might help as would answering her questions in the same way. A direct and
a tilt-in-space chair but neither of these would address the honest approach is best but must be consistent with the
immediate problems. parent’s wishes since she is a minor child.

141. medial rotation. Common abnormal postural findings 149. overwork damage in weakened, denervated muscle.
consistent with anterior knee pain in an adolescent female Amyotrophic lateral sclerosis is a progressive
include pes planus, lateral tibial torsion and genu valgum degenerative disease that affects both upper and lower
with compensation at the femur of excessive medial rotation. motor neurons. An important early goal of physical
Lateral femoral rotation is commonly observed with genu therapy is to maintain the patient's level of conditioning
varum. Retroversion of the hip is an abnormally small angle while preventing overwork damage in denervated muscle
between the femoral neck and condyles and is not affected (lower motor neuron injury). Myalgia is common in I,MN
by posture. lesions. It can be ameliorated " but not prevented. Ataxia
and radicular pain are not associated with ALS.
142. you will meet with the occupational therapist to discuss the
exercise approaches and you will let them know the 150. slight wrist extension with fingers supported and thumb in
outcome of the meeting as soon as possible. Any partial opposition and abduction. A resting splint that
discrepancies in educational approaches used with the positions the wrist and hand in a functional position includes
patient's family should be handled by the learn members, 10-20 degrees of wrist extension, fingers supported, and
not the family or rehabilitation supervisors. A team meeting is thumb in partial opposition and abduction.
a good place to iron out approaches to patient care and
ensure consistency. 151. complaints of fatigue with increasing dyspnea and cough.
Left heart failure is a condition in which blood is not
143. a videotape of another child with cerebral palsy on a Swiss adequately pumped out of the heart by the left ventricle. The
ball. A videotape of another child with CP on the ball blood; therefore, backs up creating pulmonary signs and
represents the best choice to engage this child: The other symptoms (increased pulmonary artery pressures, cough,
choices, while important educational media, are not likely to dyspnea, orthopnea), weakness, and fatigue. All other
adequately present the three-dimensional qualities of choices are signs associated with right-sided heart failure.
performance needed for exercising on the Swiss ball.
152. take a rest period and only use the device 10 times per
144. summon emergency medical services. If the victim is hour. If a patient feels lightheaded with an incentive spirometer,
unresponsive, Emergency Medical Services must be it may be that they are blowing off too much CO2 by
activated immediately by calling 911 or calling a "code" if in hyperventilating. An incentive spirometer should always be
a health care institution. If the victim is a young child or used in the most upright position possible to attain the highest
infant, try one minute of rescue breathing before values possible.
summoning help.
153. 30 minutes each day. Motor learning is enhanced by daily
145. call the girl’s pediatrician immediately. These signs could practice of moderate duration.
be the result of increased cerebral edema due to a clogged
or infected shunt. Medical attention should he obtained 154. compression of the long thoracic nerve. Vigorous upper
immediately to avoid damage to the brain. limb activities can cause inflammation of soft tissues
surrounding the shoulder resulting in compression of the long
146. protection and splinting of the abdominal musculature. thoracic nerve and weakness of the serratus anterior. The
Diastasis recti abdominis is a condition in which there is a serratus anterior stabilizes the scapula with greater than 90
lateral separation or split of the rectus abdominis. It is degrees of abduction. Supraspinatus tendinitis or weakness
important to teach protection (splinting) of the abdominal does not result in scapular winging. The supraspinatus muscle in
musculature. Patients should be instructed to avoid full sit- concert with the deltoids initiates abduction in the upper
ups or bilateral straight leg raising. Pelvic floor exercises extremity. Subdeltoid bursitis causes pain with all active range
should be done but are not remediation for diastasis of motion arid does not result in scapular winging.
recti.

’05 Sullivan A
Practice Questions 29

155. sacral sitting. Spasticity is typically strong in antigravity ambulation on level surfaces only. Therefore, the PT is not
muscles. In the lower extremities this is usually the hip and knee responsible, the PTA is responsible and negligent.
extensors, adductors, and plantar flexors. Strong extensor
tone results in sacral sitting with the pelvis tilted posteriorly. 165. gentle AROM exercises in weight bearing. Aggressive soft
tissue stretching and strengthening can promote myositis
156. peripheral edema. Peripheral edema is a symptom of ossificans. Gentle weight bearing AROM exercises to
overdosage/toxicity for patients on Premarin. The risk of patient's tolerance will minimize the chance of myositis
osteoporosis and fracture is reduced for individuals on estrogen ossificans and promote improved function.
replacement therapy. Control of hot flashes is a major indication
for its use. 166. a tone inhibiting ankle-foot orthosis (AFO). A tone inhibiting
AFO will make ambulation easier and will help maintain ROM
157. jogging is safe but you might want to switch to swimming of the affected ankle. The most appropriate assistive device
during later months. According to the American College would be a cane (i. e. straight or SBQC), not a walker.
of Sports Medicine, women can continue to exercise
regularly (3 times a week) throughout pregnancy if no 167. inadequate contraction of the ankle dorsiflexors.
additional risk factors are present. After the first trimester Weakness or delayed contraction of the ankle dorsiflexors
women should avoid exercise in the supine position since or spasticity in the ankle plantarflexors may cause foot drop
this position is associated with decreased cardiac output. during midswing. Excessive extensor synergy would cause
Non-weightbearing exercise (swimming) is an appropriate plantarflexion during stance.
alternative to decrease the risk of injury and facilitate
continuation of exercise throughout pregnancy. 168. tell her that she must have the permission of her father
before she can look at the chart. The only time
158. concentric exercises, 3 sets of 10, with gradually confidentiality can be breached is if the patient agrees or he is
increasing intensity. This patient is experiencing in danger. Even the family doesn't have the right to the medical
delayed-onset muscle soreness (DOMS) as a result information without the patient's permission unless the patient
of vigorous exercise or muscular overexertion. It is a minor child or is incompetent to make decisions.
typically begins 12 to 24 hours after exercise, peaks in
24 to 48 hours, and can last up to 5 to 7 days. DOMS 169. decrease edema. The reduction of edema is through the
is usually greater after muscle lengthening or eccentric diffusion of synovial fluid. In addition, increased venous flow and
exercise and can be lessened by gradually increasing increased nutrition is occurring in and around the joint
intensity and duration of exercise. enhancing the healing process. The reduction of pain is a
secondary consequence of decreased inflammation. Range of
159. an iliotibial band friction disorder. A positive Noble motion is usually limited during the first few hours of treatment
compression test is an indication of an i l i o t i b i a l band and no tension is to be placed on the surrounding tissues.
friction disorder. A FABERE test can distinguish between h i p
joint pathology and sacroiliac joint dysfunction. 170. five times per week. The patient with acute tendinitis receiving
iontophoresis should come daily the first two weeks to enable
160. problems with motor planning and scaling of movements. optimal amounts of medication to affect the inflamed tissue.
The basal ganglia functions to convert general motor activity The patient needs to return to work as soon as possible.
into specific, goal-directed action plans. Dysfunction results
in problems with motor planning and scaling of movements 171. hip flexion contracture. A hyperextended knee can be
and postures (e.g., bradykinesia). caused by extensor spasticity, quadriceps weakness (a
compensatory locking of the knee), or by plantarflexion
161. vagus nerve. These are the tests for the vagus nerve C.N. contractures or deformity.
X. See chapter 2 for cranial nerve tests.
172. call emergency medical technicians as it may be a
162. increased volume of air at resting end expiratory pressure pneumothorax. The combined signs and symptoms of
(REEP). The gravity independent area of the lung in the absent breath sounds, sudden onset of chest pain and
upright sitting position refers to the apices of I he lungs shortness of breath indicate a pneumothorax, especially in
which house the most air at REEP. That area has the least a 15 year-old male (growth spurt) with pathological changes
perfusion because of the effects of gravity on blood flow. The of king tissue. This should be considered an emergency
apices also have the smallest change in ventilation during the situation.
respiratory cycle because they are the most full at rest. Finally,
this area of the lung has the highest oxygenation and lowest 173. supervision of home health aides for completion of a daily
carbon dioxide content due to the small volume of blood that home exercise program. Medicare funds home health care
needs to be diffused. under Part A. Teaching of home care aides to ensure
completion of a daily home exercise program is a
163. just anterior and superior to the greater trochanter. The reimbursable service while routine provision of an
orthotic hip joint should coincide with a point just anterior and established maintenance program in which the therapist
superior to the greater trochanter (the anatomic center of the performs the exercises would not be covered.
hip joint).
174. PNF bilateral symmetrical upper extremity D2 flexion
164. the PTA who is completely liable because the plan of care patterns, rhythmic initiation. The patient with Parkinson's
was altered without communicating with the supervising disease typically develops elbow flexion, shoulder
PT. Only the supervising PT may alter the established plan of adduction contractures of the upper extremities along with
care. In this case the treatment plan stipulated progressive a flexed, stooped posture. Bilateral symmetrical upper

‘05 Sullivan A
30 Practice Questions

extremity PNF D2F patterns encourage shoulder flexion 181. referral to a physician to evaluate anterior compartment
and abduction with elbow extension, and upper trunk pressures during activity. Deep anterior leg cramping after
extension (all needed motions). Both quadruped and running distances- one mile or greater would most likely be
modified plantigrade positions encourage postural flexion. due to a local ischemic condition of the anterior
compartment as a result of excessive pressure. Shin splints
175. perform the ROM exercises yourself. The practice of using and tendinitis symptoms would occur earlier in running and
supportive personnel falls under the Guide for Professional be felt as pain. Referral to a physician is indicated if
Conduct (Appendix B0 and under the individual practice nontraumatic compartment syndrome is suspected.
acts of the states. Delegated responsibilities should be Traumatic anterior compartment syndrome requires
commensurate with the qualifications (experience, immediate attention by a physician. Spinal stenosis usually
education, training) of the individual to whom manifests itself as posterior leg cramping.
responsibilities are being assigned. In this case, it is not
reasonable to assume an aide newly arrived to the P.T. 182. teach her activity pacing and energy conservation
department has the knowledge or skills to do this techniques. You should teach this patient activity pacing
treatment. A brief orientation to ROM exercises is not and energy conservation techniques. It is important she is
adequate to ensure proper treatment. The therapist should able to balance rest with activity in order to not further
do the ROM exercises herself. In some states, any weaken muscles affected by progressive post polio
treatment by an aide would be inappropriate. muscular atrophy.

176. wheelchair with head and trunk support with abductor 183. ligamentum flavum. The ligamentum flavum becomes
wedge. The wheelchair would be the most functional hypertrophied with lumbar spondylosis and may invade the
adaptive equipment with postural support and abductor intervertebral foramen compressing the left L5 spinal nerve
wedge to help decrease extensor spasms of lower root.
extremities. This individual is not stable enough to use a
rollator walker or scooter board. 184. onset of moderate to severe angina. According to the
American College of Sports Medicine, an absolute
177. back extension, costal expansion exercises, and indication for terminating an exercise bout is onset of
maintenance of proper posture to prevent deformity. moderate to severe angina. Other absolute indications
Postural reeducation will help to prevent further increase in include: acute MI, a drop in SBP with increasing workload,
thoracic kyphosis and costal expansion exercise will serious arrhythmias (2nd or 3rd degree heart blocks,
improve breathing efficiency. Abdominal strengthening and sustained ventricular tachycardia or PVCs, atrial
stretching of scapular stabilizers are not indicated for fibrillation with fast ventricular response), unusual or
ankylosing spondylitis. severe shortness of breath, CNS symptoms (ataxia,
vertigo, confusion), or patient's request. The other choices
178. contralateral hemiparesis and sensory deficits with greater are considered relative indications and require close
involvement in the arm than the leg. A CVA affecting the monitoring.
middle cerebral artery will result in symptoms of
contralateral hemiparesis and hemi sensory deficits with 185. notify the physician immediately, you suspect heterotopic
greater involvement of the arm than the leg. Choice B is ossifications may be developing. Early signs of
incorrect since involvement would be the left hemisphere heterotopic bone formation include swelling, pain,
with right hand dominance. Choice C (greater leg erythema occurring near a large joint. The physician
involvement than arm) is characteristic of a CVA affecting the should be notified immediately. Vigorous stretching is
anterior cerebral artery. contraindicated.

179. elevated levels of low-density lipoprotein or LDL. 186. clearance between the floor and the foot plate of at least 2
Increased blood cholesterol levels with high levels of inches. The correct measure for seat height in a
low-density lipoproteins (LDLs) increases the risk of wheelchair is 2 inches clearance between the floor and the
coronary artery disease (CAD); conversely low foot plate, measured from the lowest point on the bottom
concentrations of high-density lipoproteins (HDLs) is of the footplate.
also harmful. The link between CAD and triglycerides
is not as clear. 187. recreational activities such as swimming or biking.
Recreational exercise will be both fun and helpful in
180. thank him very much, but refuse his invitation while he is maintaining functional level as long as possible. At 6 years
receiving treatment. Physical therapy departments should of age, wheelchair confinement is not usual.
have a policy stating it would be unethical to have sexual
contact or dating between staff and patients. The Guide for 188. currently there is no limit to her Medicare coverage for
Professional Conduct of the American Physical Therapy outpatient PT services. From September J, 2003 through
Association (Appendix B, Administration Chapter) states in December 7, 2003 there was a limit on the amount
Principal 2 that physical therapists shall not engage in any Medicare would pay for outpatient physical therapy
sexual relationship or activity, whether consensual or non- services. The limit of $1590.00 was in effect for outpatient
consensual with any patient while a physical therapist-patient PT services given at a private clinic but not for hospital
relationship exists. Some authorities suggest a waiting period outpatient department services. Effective December 8, 2003
of 6 months after discharge from treatment before the limit was removed by the recent Medicare Prescript ion
commencing an intimate relationship. Drug and Modernization Act of 2003. Currently there is no
limit to the amount of medically necessary outpatient PT, OT,
or SLP services.

’05 Sullivan A
Practice Questions 31

from Paget's disease are aggravated by positions where


189. use an alternate eyelid taping schedule which will allow her the lumbar spine is in extension. Because this is a chronic
to use eye movements to communicate. Pontine lesions condition, modalities are not the most effective
that result in locked-in syndrome leave the patient with an management strategy. Lumbar extension exercises decrease
inability to move or speak but with full cognitive function. By the space within the vertebral foramen thereby increasing
taping one eye open, the patient's ability to receive sensory symptoms associated with stenosis and Paget's disease.
inputs is increased (sensory deprivation is lessened) and eye
movements can be used for communication. They should 199. the posterior stop setting the foot in too much
not expect any active responses from the patient. plantarflexion. An ankle-foot orthosis can be used to assist
in knee control for the patient with hemiplegia. A foot set in
190. maintain the keyboard in a position allowing a neutral wrist slight plantarflexion (generally about 5 degrees) helps to
position. Work involving increased wrist deviation from a create an extension moment at the knee and can thus assist
neutral posture in either flexion/extension or radial/ulnar weak knee extensors in extending the knee. However if the
deviation have been associated with increased reports ankle is set in too much plantarflexion, it can cause genu
of carpal tunnel syndrome and other wrist and hand recurvatum.
problems.
200. daily walking through pain in order to attain predetermined
191. depressed ST segment with a flat T wave and shortened distances. Patients with peripheral vascular disease with
QT interval. Digitalis produces characteristic changes on intermittent claudication can benefit from a walking program
the ECG: gradual downward sloping of ST segment with a to improve peripheral blood flow and functional capacity.
flat T wave and shortened QT interval. The program should be of moderate intensity and duration.
Patients should be encouraged to exercise to the point of
192. intermittent pneumatic compression, extremity elevation, pain, not through or beyond.
and massage. Lymphedema following surgery and radiation
can be effectively managed by external compression and
extremity elevation. Manual lymph drainage (massage and
PROM) are also appropriate interventions. Exercise and
positioning alone would not provide the needed lymph
drainage; isometric exercise is contraindicated.

193. suggest that the patient speak with his surgeon. It is within
the physician's scope of practice to discuss the indications
and problems that could arise from this surgical procedure. It
is not within the physical therapist's scope of practice to be the
expert who discusses problems associated with surgery.

194. gravity assisted right upper extremity exercises to promote


scapular control following damage to the long thoracic
nerve. With axillary dissection the long thoracic nerve
may be damaged. This leads to serratus anterior
weakness and loss of scapular control. Gravity
assisted exercises to promote scapular control should
be emphasized early in rehabilitation to help restore
proper scapular humeral rhythm.

195. amount of productive secretions decreases. The purpose


of postural drainage is to help remove secretions. If the
amount diminishes this might be an indicator that the
treatment has been successful and that the frequency of
treatment can be reduced. The other choices of lever,
sputum consistency and pain do not provide a rationale
to decrease treatment frequency.

196. chronic, episodic bouts of dysfunction. Meniere's disease


is a recurrent and usually progressive vestibular disease
characterized by episodic bouts of dysfunction. As w i t h
most vestibular disorders, varying degrees of vertigo,
nausea, and postural i n s t a b i l i t y will occur.

197. Workers’ Compensation. T h e employee was injured on


the job even though she arrived early. She is therefore
covered by Worker's Compensation for her injury.

198. postural reeducation to prevent positions that increase


symptoms. Patients should be educated to prevent
extended positions for long periods. Symptoms resulting

‘05 Sullivan A

You might also like